Примеры пределов с ответами: Примеры решения пределами с ответами

Содержание

Практическое занятие №3 предел функции

Цель: формирование навыков решения задач по нахождению пределов функции с неопределенностью вида: и .

Контрольные вопросы

1. Сформулируйте понятие числовой последовательности.

2. Какая числовая последовательность называется ограниченной?

3. Какая точка называется предельной для данной числовой последовательности.

4. Сформулируйте теорему Больцано-Вейерштрассе.

5. Сформулируйте понятие предела числовой последовательности.

6. В чем заключается геометрический смысл предела числовой последовательности?

7. Сформулируйте понятие функции. Способы задания функции.

8. Сформулируйте понятие предела функции.

9. Запишите основные свойства пределов.

10. Запишите первый и второй замечательные пределы.

Примеры решения типовых задач

Пример 3.1.

Вычислить предел:

Решение:

Пример 3.2.

Вычислить предел:

Решение:

Если вместо подставить 2, то получится неопределенность вида . Чтобы избавится от нее, преобразуем числитель и знаменатель:

Числитель:

Знаменатель:

,

Таким образом:

Пример 3.3.

Вычислить предел:

Решение:

Если вместо подставить , то получится неопределенность вида . Чтобы избавится от нее разделим числитель и знаменатель на :

Пример 3.4.

Вычислить предел:

Решение:

Если вместо подставить 0, то получится неопределенность вида

Преобразуем данный предел:

Здесь использовали замену и первый замечательный предел.

Пример 3. 5.

Вычислить предел:

Решение:

Здесь выполнили замену и второй замечательный предел.

Задачи для самостоятельного решения:

3.1.

3.2. ; 3.3. ; 3.4. ; 3.5. ;

3.6.

Ответы: 3.1.

7; 3.2. -1; 3.3. 2; 3.4. 2; 3.5. ; 3.6. .

Практическое занятие №4 дифференциальное исчисление функции одной переменной

Цель: формирование навыков решения задач по нахождению производной от функции. Решения задач по исследованию функции на интервалы возрастания и убывания, а также точки максимумов и минимумов.

Контрольные вопросы

1. Сформулируйте понятие непрерывной функции в данной точке.

2. Сформулируйте понятие точки разрыва функции.

3. В чем принципиальное различие между точками разрыва первого и второго рода?

4. Сформулируйте понятие производной функции одной переменной.

5. Какая функция называется дифференцируемой в данной точке?

6. В чем состоит геометрический смысл производной?

7. Запишите правило дифференцирования суммы функций.

8. Запишите правило дифференцирования частного функций.

9. Запишите правило нахождения производной от произведения функций.

10. Сформулируйте правило дифференцирования сложных функций.

11. Сформулируйте понятие дифференциала функции одной переменной.

12. Сформулируйте понятие точек максимумов и минимумов функции.

13. Запишите необходимый и достаточный признаки существования точек экстремумов функции.

Пример 4.1.

Найти производную функции:

Решение:

Пример 4.2.

Найти производную функции: .

Решение:

Воспользуемся правилом дифференцирования произведения функций:

Пример 4. 3.

Найти производную функции:

Решение:

Воспользуемся правилом дифференцирования частного

Пример 4.4.

Найти производную сложной функции:

Решение:

Пример 4.5.

Исследовать на максимум и минимум функцию:

Решение:

Находим критические точки. Продифференцируем данную функцию:

Находим действительные корни производной:

; .

Производная всюду непрерывна, значит, других критических точек нет.

Исследуем первую критическую точку . Так как , то:

При имеем ;

При имеем .

Значит, при переходе (слева направо) через значение производная меняет знак с плюса на минус. Следовательно, при функция имеет максимум.

Исследуем вторую критическую точку :

При имеем ;

При имеем .

Значит, при переходе (слева направо) через значение производная меняет знак с минуса на плюс. Следовательно, при функция имеет минимум.

.

Пример 4.6.

Найти прямоугольный треугольник наибольшей площади, имеющий гипотенузу, равную 10 см.

Решение:

Площадь прямоугольного треугольника можно найти по формуле:

, где и длины катетов. Согласно теореме Пифагора:

, где – длина гипотенузы.

Таким образом, подставив данное выражение в формулу для нахождения площади, получим функцию одной переменной:

.

Нам необходимо найти прямоугольный треугольник наибольшей площади, т. е. по сути, нам необходимо исследовать на экстремумы функцию и найти при каком значении она максимальна.

Находим производную от функции :

Находим критические точки: , , Используя достаточный признак существования экстремума функции, можно определить, что точкой максимума будет . Тогда длина второго катета будет равна: . Таким образом, максимальная площадь будет равна:

см2.

Задачи для самостоятельного решения:

4.1. Найти производные функций:

а) ; б) ;

в) ; г) .

4.2. Найти производную сложной функции:

а) ; б) ;

в) ; г) .

4.3. Найти экстремумы функции:

а) ; б) .

4. 4. Найти наибольшее и наименьшее значения функции на отрезке .

4.5. В тюрьме города Ленинск собрались строить железную камеру для содержания особо опасных преступников. Какое наименьшее количество железа нужно для этой цели, если по санитарным нормам высота камеры должна быть не менее 2,5 м, а ее площадь — не менее 6 м2?

Ответы: 4.1.а

; 4.1б. ; 4.1в. ; 4.1г. ; 4.2а. ; 4.2б. ; 4.2в. ; 4.2г. . 4.3а. при ; 4.3б. при , при ; 4.4. Наибольшее значение при , наименьшее значение при ; 4.5. 36,5 м2

ПРАКТИЧЕСКОЕ ЗАНЯТИЕ №5

СЛУЧАЙНЫЕ СОБЫТИЯ

СЛОЖЕНИЕ И УМНОЖЕНИЕ ВЕРОЯТНОСТЕЙ

Цели: 1) Формирование основных представлений теории вероятностей: случайные события, совместные и несовместные события, вероятность и т. д. 2) Формирование навыков расчета вероятностей на основании классического определения вероятности. 3) Формирование навыков решение задач с использованием теорем сложения и умножения вероятностей.

Контрольные вопросы

1. Сформулируйте понятие случайного события.

2. Совместные и несовместные события.

3. Достоверные и невозможные события.

4. Какие события называются единственновозможными?

5. Какие события образуют полную группу? Противоположные события?

6. Классическое определение вероятности. Диапазон изменения вероятности случайного события.

7. Правило умножения в комбинаторике. Понятие факториала.

8. Перестановка, сочетание, размещение, размещение с повторениями.

9. Сумма нескольких событий.

10. Теорема сложения двух совместных событий.

11. Теорема сложения двух несовместных событий.

12. Произведение двух событий.

13. Какие события называются независимыми в совокупности?

14. Вероятность совместного появления двух зависимых событий. Условная вероятность.

15. Вероятность совместного появления двух независимых событий.

Пример 5.1.

В ящике 7 белых и 9 черных шаров. Наугад вынимают 1 шар. Найти вероятность того, что будет вынут шар черного цвета

Решение:

Пусть A – событие, состоящее в том, что вынутый черный шар.

=7+9=15 – число всех равновозможных исходов опыта.

=9 – число исходов благоприятствующих событию .

Воспользуемся формулой классической вероятности: ;

.

Пример 5.2.

В колоде 36 карт. Найти вероятность того, что вытащенная наугад карта из колоды оказалась королям (ответ:1/9).

Решение:

Пусть A – событие, состоящее в том, что из колоды вынули короля.

Воспользуемся формулой классической вероятности: .

В колоде 36 карт, значит = 36 – число всех равновозможных исходов опыта.

Поскольку в колоде всего четыре короля, то = 4 – число исходов благоприятствующих появлению события .

.

Пример 5.3.

На карточке спортлото 36 клеток. Играющий должен отметить 4. Каково число всех возможных вариантов?

Решение:

Из условия задачи следует, что необходимо найти число сочетаний 4 из 36. Воспользуемся формулой:

Пример 5.4.

В ящике 15 деталей, среди которых 10 нестандартных. Сборщик наудачу извлекает 4 детали. Найти вероятность того, что среди извлеченных деталей две будут нестандартные.

Решение:

Пусть событие – среди извлеченных 4 деталей 2 нестандартные.

– общее число четверок, которые можно сформировать из 15 деталей

– число четверок, благоприятствующих событию А.

По формуле

Пример 5.5.

Студент сдает экзамен по теории вероятностей. Вероятность получить на экзамене «2» равна 0,1; «3» – 0,6; «4» – 0,2; «5» – 0,1. Какова вероятность того, что студент получит на экзамене положительную оценку?

Решение:

Пусть событие А – студент получит на экзамене положительную оценку.

, т.к. событие А и событие «2» – получить двойку на экзамене являются противоположными.

.

Пример 5.6.

Вероятность того, что первый стрелок поразит мишень равна 0,8, второй – 0,5. Найти вероятность того, мишень будет поражена только один раз.

Решение:

Пусть событие А – попадет первый стрелок. . Событие – мишень поразит второй стрелок. . Интересующее нас событие – будет ровно одно попадание по мишени, если стрелки сделают только по одному выстрелу.

Вероятность того, что первый стрелок не попадет: . Второй стрелок не попадет с вероятностью .

Вероятность события равна:

.

Пример 5.7.

Две пушки стреляют по мишени. Вероятность поражения мишени первой пушкой равна =0,75, второй =0,6. Какова вероятность того, что мишень будет поражена, если пушки сделают по одному залпу? События и независимы.

Решение:

Интересующие нас событие – будет поражена мишень.

Мишень будет поражена либо когда будет одно попадание, либо два. Таким образом, необходимо найти вероятность хотя бы одного попадания по мишени. Воспользуемся теоремой сложения совместных событий:

Поскольку события и независимы, данную формулу перепишем в следующем виде:

.

Пример 5.8.

В урне находятся 7 белых и 3 черных шара. Подряд извлекают 2 шара. Какова вероятность того, что оба они черные.

Решение:

Пусть событие А – первый шар черный, событие В – второй шар черный.

Данная задаче решается с использованием теоремы о вероятности совместного появления двух зависимых событий.

.

Задачи для самостоятельного решения:

5.1. В урне находятся 16 белых и 8 черных шаров. Какова вероятность того, что наудачу вынутый шар будет белым?

5.2. Брошены две игральные кости. Найти вероятность того, что сумма выпавших очков равна 5.

5.3. Студент знает 20 из 25 вопросов программы. Найти вероятность того, что студент знает предложенные ему экзаменатором 3 вопроса.

5.4. Преступник знает, что шифр сейфа составлен из цифр 1, 3, 5 и 8, но не знает, в каком порядке их набирать. Какова вероятность того, что преступник откроет сейф с первой попытки?

5.5. В урне 6 белых и 4 черных шара. Из этой урны наудачу извлекли 5 шаров. Какова вероятность того, что 2 из них белые, а 3 черные?

5.6. Какова вероятность того, что в компании из человек хотя бы у двоих совпадут дни рождения? (Для простоты предполагается, что 29 февраля не является днем рождения).

5.7. Три стрелка независимо друг от друга стреляют по мишени. Вероятность попадания в цель для первого стрелка равна 0,7, для второго – 0,85, для третьего – 0,6. Определить вероятность того, что один стрелок попадет в цель.

5.8. Студент сдает два экзамена: экономику и математику. Вероятность сдачи экономики равна , а математики – . Найти вероятность того, что студент:

а) сдаст два экзамена,

б) сдаст один экзамен,

в) сдаст хотя бы один экзамен;

г) не сдаст ни одного экзамена;

д) сдаст сессию?

5. 9. Из колоды в 36 карты наугад одна за другой вынимаются две карты. Найти вероятность того, что вынули валет и короля.

5.10. Для сигнализации об аварии установлены два независимо работающих сигнализатора. Вероятность того, что при аварии сигнализатор сработает, равна 0,95 для первого сигнализатора и 0,9 для второго. Найти вероятность того, что при аварии сработает только один сигнализатор.

5.11. Из четырех человек A, B, C, D один (A) получил информацию, которую в виде сигнала «да» или «нет» сообщает второму (B), второй – третьему (С), третий – четвертому (D), а последний (D) объявляет результат полученной информации таким же образом, как и все другие. Известно, что каждый из них говорит правду только в одном случае из трех. Какова вероятность того, что первый из лгунов сказал правду, если четвертый сказал правду?

Домашнее задание: 1) Владелец одной карточки лотереи «Спортлото» (6 из 49) зачеркивает 6 номеров. Какова вероятность того, что им будет угадано все 6 номеров в очередном тираже. 2) Лекция №7 (п. 2,3).

Вычисление пределов – математика, уроки

Цели:  Предметные:

Изучить правило вычисления пределов «на бесконечности»

Развивающие:

Содействовать развитию умений использовать научные методы познания

Методы:  исследовательская деятельность, РНС оценивания результатов учебной деятельности студентов.

Тип урока: урок изучения нового материала.

Вид урока: урок-исследование

  1. < >

    Организационный момент

  2. < >

    Проверка явки студентов

  3. Заполнение журнала

  4. Проверка готовности студентов к уроку

  5. Актуализация знаний студентов

  6. Проверка имеющихся знаний и умений

  7. Подготовка к изучению новой темы.

  8. Изучение нового материала

  9. Первичное закрепление

  10. Устный счет

  11. Решение примеров в конспекте

  12. Проверка решений

  13. < >

    Домашнее задание

Время, мин.

Действия преподавателя

Действия студентов

1

3

Приветствует, проверяет явку, заполняет журнал, интересуется настроением

Отвечают на вопросы

2

17

Мы изучили несколько приемов вычисления пределов.  Прежде, чем продолжить изучение темы, повторим, что мы знаем и умеем. Какие приемы знаем?

Устно: Слайд №3

Чему равны: 1)2) 3) 4)

На чем основано вычисление этих пределов?

Что такое неопределенность?

Что значит – раскрыть неопределенность?

Какими средствами это достигается?

Отвечают

Чтобы  завершить повторение, выполните следующее задание 1: Слайд №4

Решите, выпишите ответы. Поменяйтесь тетрадями и проверим ваши решения: Слайд№5       

Решают в конспектах,

 записывают ответы,

Проверяют работы друг

 друга

3

40

Сегодня мы продолжим наполнять копилку всевозможных способов вычисления пределов. Запишите тему.Слайд№2

Мы будем сегодня исследователями. Его цель – что-нибудь открыть, новенькое.

Для этого поделимся на 3 группы (по колонкам). Каждая группа получает задание в конверте и выполняет его общими усилиями. Я назначаю в каждой группе руководителя – любите и жалуйте и совместно работайте.

По прошествии 20 минут я попрошу по 1 пределу из каждой группы вычислить на доске  и назвать остальные ответы. Дальше попробуем понять, что получилось.

Руководитель получает

 задание, студенты обсуждают, решают, как будут его выполнять.

Выполняют.

Результаты: на доске прорешали по 1 примеру, назвали остальные ответы

1 группа

2 группа

3 группа

Анализируем 1 группу, 2 группу, 3 группу

Слайды №6,№7,№8

и Сводная таблица результатов: Слайд №9

Что обращает на себя внимание во-первых? Ваше первое впечатление?

Что общего у всех пределов? У пределов 1 группы? У 2 группы? У 3 группы?

Какие отличия?

Сведем информацию в таблицу: Слайд №10

Отвечают на вопросы

Общее

Различия

Можно предположить

1. все пределы на бесконечности

2.пределы от дробно-рациональных функций

3.ответы в каждой группе не случайные

показатели степеней равны

в 1 группе

показатели степеней разные

во 2 и в 3 группах:

  Во 2 группе – у числителя показатель больше, чем у знаменателя;

В 3 группе – у числителя показатель меньше, чем у знаменателя;

1.если показатели степеней числителя и знаменателя равны, то предел равен отношению коэффициентов при этих степенях

2.если показатель степени числителя больше показателя степени знаменателя,

то предел равен бесконечности

3. если показатель степени числителя меньше показателя степени знаменателя, то предел равен 0.

Нельзя сказать, что мы доказали утверждение.                                Конспектируют, зарисовываютслайд

Мы увидели определенную закономерность.

Другие ее доказали, мы воспользуемся этим фактом.

Откройте учебники, запишите вывод в конспект.

Слайд№11:

4

20

4.1.Пользуясь полученным результатом, вычислите

пределы: Слайд №12                                                                                   Считают устно

4.2.Вычислить самостоятельно: См. Приложение 1.                             Выполняют задание

  • Выполнить задание №2 в конспекте
  • Выписать ответы

5

5

Проверка – Слайд №13.                                                                                 Взаимопроверкасослайда№13

Сформулируйте  правило вычисления «пределов на бесконечности».

Взаимопроверка со слайда №13

6

3

Самооценка

Заполняют таблицу

Я решил(а) «n» примеров в первом задании

n  (5)

Я в группе получил(а) «m» баллов из 5.  (Коллективная оценка работы каждого)

m (5)

Я понял(а) правило вычисления пределов «на бесконечности»

1   (1)

Я могу грамотно! сформулировать это правило

1   (1)

Я решил(а) «к» примеров во втором письменном задании

k   (5)

Я ответил (а) устно на «в» вопросов

в   (6)

Я все записал(а) в конспект

1   (1)

Я выполнил(а) предыдущее д/з

1   (1)

Максимальное количество баллов – 25

«5» – 23-25, «4» – 19-22, «3» – 13 – 18, «2»  <13

n+m+в+к+…

7

2

Д/З

Составить 3 примера на вычисление пределов на «бесконечности» с 3 разными ответами. (Консультация)

Записывают

 

 

Задание №2: (задание на бумажном носителе)

Приложение 1

Вычислить пределы

Вариант 1

Вычислить пределы

Вариант 2

Вычислить пределы

Вариант 3

Вычислить пределы

Вариант 4

 

Просмотр содержимого документа
«Вычисление пределов»

Разработка урока

по теме Вычисление пределов

«на бесконечности»

Автор:Баскакова Татьяна Владимировна

2015 год

Тема урока

Вычисление пределов «на бесконечности»

  1. Дидактическое обоснование урока

Цели: Предметные:

Изучить правило вычисления пределов «на бесконечности»

Развивающие:

Содействовать развитию умений использовать научные методы познания:

наблюдение, анализ, сравнение, построение гипотезы

Воспитательные:

Методическое обеспечение урока:

Кабинет, интерактивная доска, мультимедийная установка.

Слайды, задание для проверки первичного усвоения знаний, задания для проведения исследования, карта самооценки.

Методы: исследовательская деятельность, РНС оценивания результатов учебной деятельности студентов.

Тип урока: урок изучения нового материала.

Вид урока: урок-исследование

  1. План

  1. Организационный момент

  1. Актуализация знаний студентов

  1. Изучение нового материала

  2. Первичное закрепление

  • Устный счет

  1. Проверка решений

  2. Самооценка

  3. Домашнее задание

Время, мин.

Действия преподавателя

Действия студентов

1

3

Приветствует, проверяет явку, заполняет журнал, интересуется настроением

Отвечают на вопросы

2

17

Мы изучили несколько приемов вычисления пределов. Прежде, чем продолжить изучение темы, повторим, что мы знаем и умеем. Какие приемы знаем?

Устно: Слайд №3

Чему равны: 1)2) 3) 4)

На чем основано вычисление этих пределов?

Что такое неопределенность?

Что значит – раскрыть неопределенность?

Какими средствами это достигается?

Отвечают

Чтобы завершить повторение, выполните следующее задание 1: Слайд №4

Решите, выпишите ответы. Поменяйтесь тетрадями и проверим ваши решения: Слайд№5

Решают в конспектах,

записывают ответы,

Проверяют работы друг

друга

3

40

Сегодня мы продолжим наполнять копилку всевозможных способов вычисления пределов. Запишите тему.Слайд№2

Мы будем сегодня исследователями. Его цель – что-нибудь открыть, новенькое.

Для этого поделимся на 3 группы (по колонкам). Каждая группа получает задание в конверте и выполняет его общими усилиями. Я назначаю в каждой группе руководителя – любите и жалуйте и совместно работайте.

По прошествии 20 минут я попрошу по 1 пределу из каждой группы вычислить на доске и назвать остальные ответы. Дальше попробуем понять, что получилось.

Руководитель получает

задание, студенты обсуждают, решают, как будут его выполнять.

Выполняют.

Результаты: на доске прорешали по 1 примеру, назвали остальные ответы

1 группа

2 группа

3 группа

Анализируем 1 группу, 2 группу, 3 группу

Слайды №6,№7,№8

и Сводная таблица результатов: Слайд №9

Что обращает на себя внимание во-первых? Ваше первое впечатление?

Что общего у всех пределов? У пределов 1 группы? У 2 группы? У 3 группы?

Какие отличия?

Сведем информацию в таблицу: Слайд №10

Отвечают на вопросы

Общее

Различия

Можно предположить

1. все пределы на бесконечности

2.пределы от дробно-рациональных функций

3.ответы в каждой группе не случайные

показатели степеней равны

в 1 группе

показатели степеней разные

во 2 и в 3 группах:

Во 2 группе – у числителя показатель больше, чем у знаменателя;

В 3 группе – у числителя показатель меньше, чем у знаменателя;

1.если показатели степеней числителя и знаменателя равны, то предел равен отношению коэффициентов при этих степенях

2.если показатель степени числителя больше показателя степени знаменателя,

то предел равен бесконечности

3. если показатель степени числителя меньше показателя степени знаменателя, то предел равен 0.

Нельзя сказать, что мы доказали утверждение. Конспектируют, зарисовываютслайд

Мы увидели определенную закономерность.

Другие ее доказали, мы воспользуемся этим фактом.

Откройте учебники, запишите вывод в конспект.

Слайд№11:

4

20

4.1.Пользуясь полученным результатом, вычислите

пределы: Слайд №12 Считают устно

4.2.Вычислить самостоятельно: См. Приложение 1. Выполняют задание

5

5

Проверка – Слайд №13. Взаимопроверкасослайда№13

Сформулируйте правило вычисления «пределов на бесконечности».

Взаимопроверка со слайда №13

6

3

Самооценка

Заполняют таблицу

Я решил(а) «n» примеров в первом задании

n (5)

Я в группе получил(а) «m» баллов из 5. (Коллективная оценка работы каждого)

m (5)

Я понял(а) правило вычисления пределов «на бесконечности»

1 (1)

Я могу грамотно! сформулировать это правило

1 (1)

Я решил(а) «к» примеров во втором письменном задании

k (5)

Я ответил (а) устно на «в» вопросов

в (6)

Я все записал(а) в конспект

1 (1)

Я выполнил(а) предыдущее д/з

1 (1)

Максимальное количество баллов – 25

«5» – 23-25, «4» – 19-22, «3» – 13 – 18, «2»

n+m+в+к+…

7

2

Д/З

Составить 3 примера на вычисление пределов на «бесконечности» с 3 разными ответами. (Консультация)

Записывают

Задание №2: (задание на бумажном носителе)

Приложение 1

Вычислить пределы

Вариант 1

Вычислить пределы

Вариант 2

Вычислить пределы

Вариант 3

Вычислить пределы

Вариант 4


Математический анализ в примерах и задачах. Предел функции — Образовательная платформа «Юрайт». Для вузов и ссузов.

  • Скопировать в буфер библиографическое описание

    Максимова, О. Д.  Математический анализ в примерах и задачах. Предел функции : учебное пособие для вузов / О. Д. Максимова. — 2-е изд., стер.  — Москва : Издательство Юрайт, 2019. — 200 с. — (Университеты России). — ISBN 978-5-534-07222-8. — Текст : электронный // Образовательная платформа Юрайт [сайт]. — URL: https://urait.ru/bcode/442137 (дата обращения: 03.10.2022).

  • Добавить в избранное

2-е изд., стер. Учебное пособие для вузов

  • Нравится
  • Посмотреть кому понравилось
  • Поделиться
    • Описание
    • Программа курса
    • Видео: 1
    • Выбор редакции
    • Нет в мобильном приложении
    Ознакомиться
    • Аннотация
    • Программа курса
    • Медиаматериалы 1
    • Комплекты 1

    Настоящее пособие вторая часть цикла «Математический анализ в примерах и задачах». Книга предназначена для активного использования, содержит ответы на вопросы, возникающие при изучении темы «Предел функции», и облегчает воспри-ятие новых понятий. Приводимый в начале каждого параграфа теоретический материал иллюстрируется оригинальными примерами и контрпримерами. Для закрепления основных навыков пособие включает более 110 теоретических задач и содержит 36 проверочных тестов для самостоятельного контроля и оценки уровня по-нимания материала. В последнем параграфе весь рассматриваемый материал пред-ставлен в виде обобщающих таблиц и схем.

    Практическая работа по теме Вычисление пределов функций

    Практическая работа

    Тема: «Вычисление пределов функции»

    Цель: сформировать умение находить пределы функций, использовать замечательные пределы для нахождения пределов.

    Теоретические сведения к практической работе.

    Число А называют пределом функции f(x) при (и пишут ), если для любого найдется число зависящее от , такое, что для всех , удовлетворяющих условию , выполняется неравенство

    Теоремы о пределах:

    1. (c=const).

    2. Если то:

    Первый замечательный предел:

    Второй замечательный предел (число е = 2,718…):

    или

    Замечательные пределы:

    Примеры решения:

    Когда дан любой предел, сначала просто пытаемся подставить число в функцию

    1)Пределы с неопределенностью вида  и метод их решения

    1) деление на х в старшей степени:

    Пример 1:

    Сначала мы смотрим на числитель и находим  в старшей степени:

    Старшая степень в числителе равна двум.

    Теперь смотрим на знаменатель и тоже находим   в старшей степени:

    Старшая степень знаменателя равна двум.

    Затем мы выбираем самую старшую степень числителя и знаменателя: в данном примере они совпадают и равны двойке.

    Итак, метод решения следующий: для того, чтобы раскрыть неопределенность  необходимо разделить числитель и знаменатель на  в старшей степени.

    Разделим числитель и знаменатель на 

    П ример 2:

    Найти предел

    Снова в числителе и знаменателе находим   в старшей степени:

    Максимальная степень в числителе: 3
    Максимальная степень в знаменателе: 4
    Выбираем наибольшее значение, в данном случае четверку.

    Разделим числитель и знаменатель на 

    П ример 3

    Найти предел

    Разделим числитель и знаменатель на 

    при раскрытии неопределенности вида   у нас может получиться конечное число, ноль или бесконечность.

    2. Пределы с неопределенностью вида  и метод их решения

    1) разложение числителя и знаменателя на множители.

    Пример 4

    Разложим числитель и знаменатель на множители

    Для того чтобы разложить числитель на множители, нужно решить квадратное уравнение:

    Сначала находим дискриминант:

    И квадратный корень из него: Далее находим корни: Таким образом:

    Всё. Числитель на множители разложен.

    Знаменатель. Знаменатель   уже является простейшим множителем, и упростить его никак нельзя.

    можно сократить на  :

    Теперь и подставляем -1 в выражение, которое осталось под знаком предела:

    2) умножение числителя и знаменателя на сопряженное выражение.

    П ример 5

    Найти предел

    Умножаем числитель и знаменатель на сопряженное выражение:

    Применяем вверху формулу  : Неопределенность   не пропала (попробуйте подставить тройку), да и корни тоже не исчезли. Но с суммой корней всё значительно проще, ее можно превратить в постоянное число. Как это сделать? Да просто подставить тройку под корни:

    Число, как уже отмечалось ранее, лучше вынести за значок предела.

    Теперь осталось разложить числитель и знаменатель на множители и сократить «виновников» неопределённости, ну а предел константы – равен самой константе:

    Решение данного примера в чистовом варианте выглядит так:

    Умножим числитель и знаменатель на сопряженное выражение.

    3) использование 1-го замечательного предела

    П ример 6

    Найти предел 

    Выражение под знаком предела похоже на первый замечательный предел, но это не совсем он, под синусом находится  , а в знаменателе  .В подобных случаях первый замечательный предел нам нужно организовать самостоятельно, используя искусственный прием. Ход рассуждений может быть таким: «под синусом 7х, значит, в знаменателе тоже нужно получить 7х». 
    А делается это очень просто:

    Пример 7

    Найти предел 


    Пример 8

    Найти предел

    Пример 9

    Найти предел

    Пример 10

    Найти предел

    Второй замечательный предел

    В теории математического анализа доказано, что:

    Данный факт носит название второго замечательного предела.

    Справка:  – это иррациональное число.

    В качестве параметра   может выступать не только переменная  , но и сложная функция. Важно лишь, чтобы она стремилась к бесконечности.

    Пример 11

    Найти предел 

    Данная неопределенность как раз и раскрывается с помощью второго замечательного предела. Но, как часто бывает, второй замечательный предел нужно искусственно организовать. Рассуждать можно следующим образом: в данном примере параметр  , значит, в показателе тоже нужно организовать   . Для этого возводим основание в степень  , и, чтобы выражение не изменилось – возводим в степень  :

    страшная степень превратилась в симпатичную букву  :

    При этом сам значок предела перемещаем в показатель:

    Содержание практической работы.

    1. Вычислить пределы функции:





    1. Вычислить пределы функций, используя замечательные пределы.





    Замечательные пределы с экспонентой и логарифмом

    Замечательные пределы с экспонентой и логарифмом
    Случайная страница | ТОМ-1 | ТОМ-2 | ТОМ-3
    АрхитектураБиологияГеографияДругоеИностранные языки
    ИнформатикаИсторияКультураЛитератураМатематика
    МедицинаМеханикаОбразованиеОхрана трудаПедагогика
    ПолитикаПравоПрограммированиеПсихологияРелигия
    СоциологияСпортСтроительствоФизикаФилософия
    ФинансыХимияЭкологияЭкономикаЭлектроника
    Сравниваем старшие степени: , следовательно, числитель более высокого порядка роста, чем знаменатель, и сразу можно сказать, что предел будет равен бесконечности. | Метод замены переменной в пределе | Бесконечно малые функции. Сравнение бесконечно малых | Может ли функция быть бесконечно малой на бесконечности? | Сравнение бесконечно малых функций | Что принципиально важно во всех рассмотренных примерах? | Замечательные эквивалентности в пределах | Первое правило Лопиталя | Второе правило Лопиталя | Показательная функция, с основанием, бОльшим единицы ( и т.д.) более высокого порядка роста, чем степенная функция с положительной степенью. |
    Читайте также:
    1. II. Предметы ведомства и пределы власти волостного суда
    2. II. Предметы ведомства и пределы власти губернского присутствия
    3. Вопрос № 24. Порядок рассмотрения надзорных жалоб и представлений. Пределы прав суда надзорной инстанции. Решение суда надзорной инстанции.
    4. ВОПРОС: Происходит ли выход за пределы (трансценденция) с раскрытием сахасрары?
    5. Глава 10. Пределы Некроманта
    6. Глава 36-4. Производство по заявлению о выдворении иностранца или лица без гражданства за пределы Республики Казахстан за нарушение законодательства Республики Казахстан
    7. Глава XXXII. ПРЕДЕЛЫ МЕНТАЛЬНОГО

    На практике чаще встречаются пределы и особенно их частные случаи . Предела лично ни разу не видел, а может быть, и видел, да не помню.

    Иногда перечисленные пределы называют третьим, четвёртым и пятым замечательным пределом, но своё негативное отношение к избыточной нумерации я уже высказал на урокеПравила Лопиталя, поэтому пусть это будут «просто» замечательные пределы без номеров.

    Сама техника решения мало чем отличатся от первого замечательного предела:

    Пример 10

    Найти предел, не пользуясь правилом Лопиталя

    Чтобы использовать замечательный предел необходимо применить уже знакомый искусственный приём: в знаменателе умножаем и делим на 2:

    Вот и всё. Напоминаю, что в качестве параметра «альфа» может выступать не только переменная «икс», но и сложная функция, лишь бы она стремилась к нулю. В рассмотренном примере .

    Короткий закусочный предел для самостоятельного решения:

    Пример 11

    Найти предел, не пользуясь правилом Лопиталя

    Заметьте, что условие задачи не ограничивает нас в выборе действий, примеры можно было решить и через замечательные эквивалентности:
    (эквивалентность ).


    (эквивалентности )

    Какой способ выбрать? Рекомендую всё-таки решать через замечательные пределы (конечно, если пример не дико сложный) – выглядит солиднее.

    Существенная особенность пределов состоит в том, что при перестановке числителя и знаменателя результаты тоже «переворачиваются»:

    Пример 12

    Найти предел, не пользуясь правилом Лопиталя

    Как говорится, мал пример да заковырист….

    Решаем:

    На первом шаге нужно перейти к новой переменной ТАК, чтобы она стремилась к нулю.
    Проведём замену: , тогда:
    Если , то

    Для самостоятельного решения:

    Пример 13

    Найти предел, не пользуясь правилом Лопиталя

    Если возникли затруднения на начальном этапе, пожалуйста, вернитесь к Примеру №9.

    Разберём напоследок что-нибудь посложнее. Типовой и довольно распространённый предел:

    Пример 14

    Найти предел, не пользуясь правилом Лопиталя



    Сначала полное решение, потом комментарии:

    (1) На первых шагах избавляемся от синуса. Умножим числитель и знаменатель на .
    (2) Используем первый замечательный предел , где . Константу выносим из предела.
    (3) Проводим искусственное преобразование числителя. Возьмите его на заметку, разность экспонент раскручивается именно так.
    (4) Почленно делим числитель на знаменатель.
    (5) Числитель и знаменатель первой дроби умножаем на 2. Числитель и знаменатель второй дроби умножаем на –3.
    (6) В обеих дробях используем замечательный предел , после чего остались от козлика рожки да ножки.

    Используя правило Лопиталя, выполним проверку:

    Заключительный пример посвящен раритету . Если его не встречал я, то это не значит, что его не встретите вы. Встретите. Причём, прямо сейчас =)

    Пример 15

    Найти предел, не пользуясь правилом Лопиталя

    Это пример для самостоятельного решения.

    Всего примеров получилось таки 15-ть, а не 20-ть, и ваш покорный слуга постарался отобрать самые распространенные вещи. Желающие ознакомиться с другими пределами, могут закачать соответствующий архив решений в банке задач по высшей математике. Однако должен предупредить, будьте осторожнее – некоторые экземпляры не то, чтобы сильно сложные, но точно рождены воспалённым сознанием. Я постарался разобрать тему без навороченных нелепых примеров, поскольку убеждён, что студент должен мучиться с удовольствием =)

    И приснится вам сегодня правило Лопиталя =)

    Решения и ответы:

    Пример 2

    Разложим числитель и знаменатель на множители.
    В числителе используем формулу суммы кубов :

    Знаменатель:

    Таким образом:

    Пример 4

    Умножим числитель и знаменатель на сопряженные выражения.

    Разложим числитель и знаменатель на множители:

    Пример 6

    Умножим числитель и знаменатель на сопряженное выражение, используем формулу разности кубов :

    Пример 8
    Используем формулу :

    Проведём замену переменной:
    Если , то

    Используем тригонометрическую формулу :

    Используем формулы половинного аргумента :


    Дата добавления: 2015-07-25; просмотров: 338 | Нарушение авторских прав


    | следующая страница ==>
    Вычислить предел, используя правило Лопиталя| Пример 11

    mybiblioteka. su – 2015-2022 год. (0.019 сек.)

    Пределы – Расчет 2

    Все ресурсы Расчет 2

    9 Диагностические тесты 308 практических тестов Вопрос дня Карточки Learn by Concept

    ← Предыдущая 1 2 3 4 5 6 7 8 9 … 51 52 Следующая →

    Исчисление 2 Помощь » Пределы

    Оценка:

    Возможные ответы:

    Предел не существует.

    Ни один из других ответов не является правильным.

    Правильный ответ:

    Ни один из других ответов не является правильным.

    Объяснение:

    Оценено в , числитель и знаменатель равны 0, как показано ниже:

    Таким образом, прямая замена не работает. Здесь сработает правило Лопиталя, но проще всего отметить, что

     и .

    Таким образом, выражение можно переписать и решить следующим образом:

    Report an Error

    Evaluate the limit:

    Possible Answers:

    Does Not Exist

    Correct answer:

    Объяснение:

    Прямая оценка предела даст неопределенный ответ .

    Переписывая предел в терминах синуса и косинуса, мы можем попытаться манипулировать функцией, чтобы использовать свойство .

    Умножая функцию на аргументы функции синуса, мы видим, что предел будет равен .

     

     

     

    Сообщить об ошибке

    Оценить .

    Возможные ответы:

    Предел не существует.

    Правильный ответ:

    Объяснение:

    и

    ,

    , поэтому мы не можем решить эту проблему путем замены.

    Однако мы можем переписать выражение:

    Сообщить об ошибке

    Найти предел при приближении к бесконечности.

    Возможные ответы:

    Неубедительно

    Правильный ответ: 9005

    5

    5

    Объяснение:

    Выражение  можно переписать как .

    Вспомним, что теорему сжатия можно использовать для нахождения предела. Функция синуса имеет диапазон от , что означает, что диапазон должен находиться внутри этой границы.

    Умножить член на.

    Берем предел по мере приближения к бесконечности для всех членов.

    Так как левый и правый концы этого интервала равны нулю, то можно сделать вывод, что они также должны стремиться к нулю.

    Правильный ответ: 0.

    Сообщить об ошибке

    Определить предел.

    Возможные ответы:

    Правильный ответ:

    Объяснение:

    Чтобы определить, постройте график функции и обратите внимание на направление слева и справа от кривой по мере ее приближения.

    Как влево, так и вправо уходит в минус бесконечность.

    Ответ:  

    Сообщить об ошибке

    Что из следующего верно?

    Возможные ответы:

    Если и , то  существует.

    Если существует, то и оба существуют.

     и  существовать тогда и только тогда, когда  существует.

    Если ни   , ни  не существует, то также не существует.

    Правильный ответ:

    Если и , то  существует.

    Объяснение:

    Если  и , то  существует.

    Это можно строго доказать, используя определение предела, но, скорее всего, это выходит за рамки вашего класса.

    Сообщить об ошибке

    Определить лимит:  

    Возможные ответы:

    Правильный ответ:

    Объяснение:

    Изолируйте константу в пределе.

    Свойство limit .

    Следовательно:

    Сообщить об ошибке

    Оцените предел, если возможно:  

    Возможные ответы:

    Правильный ответ: 9005 6

    5

    5 Объяснение:

    Чтобы оценить , обратите внимание, что внутренний член  будет приближаться к бесконечности после подстановки. Арктангенс очень большого числа приближается к .

    Ответ .

    Сообщить об ошибке

    Оцените следующий предел:

    Возможные ответы:

    Правильный ответ:

    Объяснение:

    Первый шаг – вынести член высшей степени из полинома сверху и снизу (фактически выделив 1):

    , что станет

    Оценивая предел, мы приближаемся .

     

    Сообщить об ошибке

    Оцените следующий предел:

    Возможные ответы:

    Правильный ответ:

    6 Объяснение:

    Чтобы оценить предел, сначала вытяните наибольший член степени сверху и снизу (по сути, мы удаляем 1):

    , что становится

    Подставляя бесконечность, мы находим, что числитель приближается к нулю, что приближает весь предел к 0,

     

    Сообщить об ошибке

    ← Назад 1 2 3 4 5 6 7 8 9 … 51 52 Далее →

    Уведомление об авторских правах

    Все ресурсы Calculus 2

    9 Диагностические тесты 308 практических тестов Вопрос дня Карточки Learn by Concept

    Определение, типы, использование и примеры, математика

    Ограничения: определение, типы, использование и примеры, математика

    Выберите ваш язык

    Рекомендуемые языки для вас:

    Европа

    английский (DE) английский (Великобритания)

    StudySmarter — универсальное учебное приложение.

    4.8 • Рейтинг +11k

    Более 3 миллионов загрузок

    Бесплатно

    Сохранять

    Распечатать

    Редактировать

    Ограничения

    СОДЕРЖАНИЕ :

    ОГЛАВЛЕНИЕ

      Вы когда-нибудь слышали поговорку « закрыть только счет в подковы и ручные гранаты »? Что ж, оказывается, это не совсем так. Близко, или почти достигает цель, также учитывается в исчислении – при работе с ограничивает , то есть!

      Основная концепция предела в математике

      Основная концепция предела в математике необходима для вашего понимания исчисления.

      Эта концепция существует уже тысячи лет; ранние математики использовали эту концепцию, например, чтобы находить все более и более точные приближений площади круга .

      Однако формальное определение предела появилось лишь с 19 века.{2}} \]

      Обратите внимание на поведение этих графиков при приближении к значению \( x=2 \).

      Обратите внимание на график, где \( x = 2 \).

      Обратите внимание на график, где \( x = 2 \).

      Обратите внимание на график, где \( x = 2 \).

      Графики этих функций показывают их поведение при \( x=2 \) и около него. Понаблюдав за ними, сможете ли вы увидеть, что у них общего?

      Все они не определены, когда \( x=2 \)!

      • Но если это все, что вы о них говорите, вы не получаете много информации, не так ли? Если вам предоставлена ​​только эта информация, то, насколько вам известно, все три функции могут выглядеть одинаково. Однако, судя по их графикам, вы понимаете, что это не так. 2-4}{x-2} \) ведет себя вблизи \( x = 2 \). Обратите внимание, что по мере того, как значения \( x \) приближаются к \ ( 2 \) с любой стороны \ ( 2 \), значения \ ( f (x) \) приближаются к \ ( 4 \).

        • Чтобы сформулировать этот факт в математических терминах, вы бы сказали: «Предел \( f(x) \) при приближении \( x \) к \( 2 \) равен \( 4 \)».

        • Это утверждение представлено в математической записи как:

        \[ \lim_{x \to 2} f(x) = 4. \]

        Отсюда вы можете начать развивать интуитивное определение предела – думая о пределе функции при числе \( a \) как о действительном числе \( L \), к которому функциональные значения приближаются по мере приближения его \( x \)-значений \( a \ ), при условии, что число \( L \) существует. Более формально это можно записать так:

        Пусть \(f(x)\) — функция, определенная при всех значениях открытого интервала, содержащего \( a \) (возможно, кроме \( a \)), и пусть \( L \) — вещественная количество. Если , то все значений \( f(x) \) приближаются к действительному числу \( L \) так же, как значения \( x \) – кроме \( x = a \) – приближаются к числу \( a \) , то можно сказать, что предел \( f(x) \) при приближении \( x \) к \( a \) равен \( L \) .

        Или, проще говоря:

        По мере того, как \( x \) все ближе и ближе к \( a \), \( f(x) \) становится все ближе и ближе и остается близким к \( L \).

        Идея предела представлена ​​с помощью математической записи как:

        \[ \lim_{x \to a} f(x) = L \]

        Дело в том, как работают ограничения! Чтобы разработать и понять ключевые аспекты исчисления, вам сначала нужно освоиться с ограничениями и тем фактом, что приближения — или приближение к желаемому значению — являются основой исчисления. Итак, теперь вы можете изменить поговорку с:

        • « закрыть только подсчеты в подковы и ручные гранаты » на
        • « закрыть только подсчеты в подковы, ручные гранаты и исчисление»!

        Нахождение пределов

        Прежде чем погрузиться в алгебраические методы, следующий шаг, который следует сделать интуитивно, — разработать способ нахождения пределов путем их оценки . Это можно сделать одним из двух способов:

        1. Решение ограничения с помощью таблицы функциональных значений

        2. Решение ограничения с помощью графика

        Решение предела с помощью таблицы функциональных значений

        Чтобы решить предел с помощью таблицы функциональных значений, вы можете использовать эту стратегию решения проблем.

        Стратегия – Решение предела с помощью таблицы функциональных значений

        1. Если вы хотите решить предел: \( \lim_{x \to a} f(x) \), вы начинаете с составления таблицы функциональные ценности.
        2. Затем посмотрите на значения в каждом из столбцов с пометкой \( f(x) \).
          • Определите, приближаются ли значения к одному значению при перемещении вниз по каждому столбцу.
        3. Если оба столбца приближаются к общему значению, то можно сказать, что\[ \lim_{x \to a} f(x) = L. \]

        Решение предела с помощью графика

        Можно расширить описанная выше стратегия решения проблем для решения предела с использованием графа.

        Стратегия – определение предела с помощью графика

        1. Следуя приведенной выше стратегии, вы можете подтвердить свой результат, построив график функции.
        2. С помощью графического калькулятора (или другого программного обеспечения) нарисуйте график рассматриваемой функции.
          • Убедитесь, что функциональные значения \( f(x) \) для значений \( x \) рядом с \( a \) находятся в графическом окне.
        3. Двигайтесь по графику функции и проверяйте \( y \)-значения, когда соответствующие им \( x \)-значения приближаются к \( a \).
          • Если \( y \)-значения приближаются к \( L \) так же, как \( x \)-значения приближаются к \( a \) с обоих направлений, то\[ \lim_{x \to a} f(x ) = L. \]

        Обратите внимание, что может потребоваться увеличить график и повторить эти шаги несколько раз.

        Дополнительные сведения и примеры см. в статьях о нахождении пределов и нахождении пределов с помощью графика или таблицы.

        Типы ограничений

        Несмотря на то, что два приведенных выше метода интуитивно понятны, они неэффективны и основаны на слишком большом количестве предположений для выполнения работы. Но как вы можете пройти мимо этих методов?

        Ну, вам нужно изучить методы решения или оценки пределов, которые по своей природе более алгебраичны.

        И как вы можете это сделать? Во-первых, вы должны знать о двух специальных ограничениях; они обеспечивают основу алгебраических методов решения пределов.

        А что такого особенного в этих двух ограничениях? Эти два предела также известны как основных пределов , поскольку они обеспечивают основу для законов пределов. Когда вы смотрите на графики ниже, что вы замечаете?

        Независимо от того, где на линии \( y = x \) находится точка \( (a, a) \), предел при приближении \(x\) к \(a\) всегда равен \(a\).

        Независимо от того, где на линии \( y = c \) находится точка \( (x, c) \), предел при приближении \(x\) к любому вещественному числу \(a\) всегда равен \(c \).

        Основываясь на этих графиках, вы можете алгебраически расписать предел этих функций. Их алгебраические интерпретации резюмируются в следующей теореме.

        Теорема: основные пределы

        Пусть \( a \) – действительное число. Пусть \(с\) — константа. Затем:

        \[ \begin{align}1. \; & \lim_{x \to a} x = a \\2. \; & \lim_{x \to a} c = c\end{align} \]

        Об этих двух пределах можно наблюдать следующее:

        1. делает \(f(x)\). 9Таблица ( \bf{ x } \) \( \bf{ f(x) = c } \) \( \bf{ x } \) \( \bf{ f(x) = c } \) \( а – 0,1 \) \( в \) \( а + 0,1 \) \( в \) 90 0 0 9.1 а – 0,10 \( в \) \( а + 0,01 \) \( в \) \( а – 0,001 \) \( в \) 5 8,93 90 а \) \( c \) \( a – 0.0001 \) \( c \) \( a + 0.0001 \) \( c \)
          • Обратите внимание, что для всех значений \( x \) – независимо от того, приближаются они к \ ( a \) или нет – значения \ ( f ( x ) \) остаются постоянными в \ ( c \).
          • Следовательно, \( \lim_{x \to a} c = c \)

        Правила ограничения

        Основываясь на этих первых двух основных правилах ограничения, правила ограничения (также называемые законами ограничения) перечислены ниже.

        Теорема: Предельные законы

        Пусть \( f(x) \) и \( g(x) \) определены для всех \( x \neq a \) на открытом интервале, содержащем \( a \). Предположим, что \( L \) и \( M \) – действительные числа, такие что:

        \[ \lim_{x \to a} f(x) = L \]

        и\[ \lim_{x \ к а} g(x) = M \]

        Пусть \( c \) — константа. Тогда верны следующие утверждения:

        Закон сумм для пределов :

        \[ \lim_{x \to a} (f(x) + g(x)) = \lim_{x \to a} f(x ) + \lim_{x \to a} g(x) = L + M \]

        Разностный закон для пределов :

        \[ \lim_{x \to a} (f(x) – g(x )) = \lim_{x \to a} f(x) – \lim_{x \to a} g(x) = L – M \]

        Постоянный кратный закон для пределов :

        \[ \lim_ {x \to a} (c \cdot f(x)) = c \cdot \lim_{x \to a} f(x) = cL \] 9{n} \mbox{ для каждого положительного целого числа } n \]

        Корневой закон для пределов :

        \[ \lim_{x \to a} \sqrt[n]{f(x)} = \sqrt[ n]{\lim_{x \to a} f(x)} = \sqrt[n]{L} \mbox{ для всех } L \mbox{, если } n \mbox{ нечетно, и для } L \geq 0 \mbox{ если } n \mbox{ четно} \]

        Имейте в виду, что существуют и другие предельные законы – теорема сжатия и теорема о промежуточном значении. Пожалуйста, обратитесь к этим статьям для получения дополнительной информации.

        Наличие предела – когда предела не существует?

        При работе со следующим примером помните, что для существования предела функциональные значения должны приближаться к одному вещественному значению; в противном случае предела не существует.

        Оценка несуществующего предела (DNE) Из-за колебаний

        Попытаться оценить

        \[ \lim_{x \to 0} sin \left( \frac{1}{x} \right ) \]

        с использованием таблицы функциональных значений.

        Решение :

        1. Создайте таблицу значений.
          \(\bf{x}\) \(\bf{sin\left(\frac{1}{x}\right)}\) \(\bf{x}\) \(\bf{sin\left(\frac{1}{x}\right)}\)
          \(-0,1\) \(0,54402\) \(0,1\) \(-0.54402\)
          \(-0.01\) \(0.50636\) \(0.01\) \(-0. 50636\)
          \(-0.001\ ) \(-0,82688\) \(0,001\) \(0.82688\)
          \(-0.0001\) \(0.30561\) \(0.0001\) \(-0.30561\)
          \(-0.00001\) \(-0.03575\) \(0.00001\) \(0.03575\)
          \(-0.000001\) \(0.34999\) \(0.000001\) \ (-0,34999\)
        2. Внимательно изучите таблицу. Что ты заметил?
          • Значения \( y \) не приближаются ни к какому значению. Таким образом, кажется, что этот предел не существует. Однако, прежде чем прийти к такому выводу, вы должны применить системный подход.
            1. Рассмотрим следующие \( x \)-значения для этой функции, приближающиеся к \( 0 \):\[ \frac{2}{\pi}, \frac{2}{3\pi}, \frac{2 }{5\pi}, \frac{2}{7\pi}, \frac{2}{9\pi}, \frac{2}{11\pi}, \cdots \]
            2. Их соответствующие \( y \)-значения: \[ 1, -1, 1, -1, 1, -1, \cdots\]
        3. На основании результатов можно с уверенностью заключить, что предела не существует . Математический способ записать это так: \[ \lim_{x \to 0} sin \left( \frac{1}{x} \right) \, DNE \]Где DNE означает «не существует».
        4. Конечно, всегда полезно построить график функции, чтобы подтвердить свой результат. График \( f(x) = sin \left( \frac{1}{x} \right) \) показывает, что функция все более и более резко колеблется между \(-1 \) и \(1 \) как \(x\) приближается к \(0\).

        Предел : \( \lim_{x \to 0} sin \left( \frac{1}{x} \right) \) не существует потому что функция сильно колеблется как \( x \ ) приближается к пределу \( 0 \).

        Односторонние ограничения

        Бывают случаи, когда утверждение, что предел функции не существует в какой-либо точке, не дает достаточной информации об этой точке. Чтобы убедиться в этом, еще раз взгляните на вторую функцию из начала этой статьи.

        \[ g(x) = \frac{|x-2|}{x-2} \]

        При выборе значений \( x \), которые все ближе и ближе к \( 2 \), \ ( g(x) \) приближается не к одному значению , а к двум значениям. Следовательно, предела не существует, т. е.

        \[ \lim_{x \to 0} g(x) \, DNE. \]

        Хотя это утверждение верно, не могли бы вы сказать, что оно не дает полной картины поведения \( g(x) \) при \( x = 2 \)?

        С односторонними ограничениями можно дать более точное описание поведения этой функции при \( x = 2 \).

        • Для всех значений \( x \) до осталось от \( 2 \) – или отрицательная часть \( 2 \) – \( g(x) = -1 \).

          • Итак, вы говорите, что , когда \( x \) приближается \( 2 \) слева, \( g(x) \) приближается \( -1 \) 9{+}} h(x) = +\infty \]

            Важно понимать, что когда вы говорите, что предел бесконечен, это не означает, что предел существует. Это просто более наглядный способ сказать, что предела не существует. \( \pm \infty \) не является действительным числом, поэтому любой бесконечный предел не является пределом, который существует.

            В общем, пределы на бесконечности определяются как:

            Три типа бесконечных пределов

            1. Бесконечный предел слева открытый интервал \( (b, a) \). 9{+}} f(x) = -\infty. \]
          • Двусторонний бесконечный предел : Пусть \( f(x) \) определено для всех \( x \neq a \) в открытом интервале, содержащем \( a \).
            1. Если значения \( f(x) \) неограниченно возрастают по мере того, как значения \( x \) (где \( x \neq a \)), приближаются к числу \( a \), то предел когда \( x \) приближается к \( a \) положительная бесконечность. Это записывается как: \[ \lim_{x \to a} f(x) = +\infty. \]
            2. Если значения \( f(x) \) неограниченно убывают по мере того, как значения \( x \) (где \( x \neq a \)), приближаются к числу \( a \), то предел при приближении \(x\) к \(a\) равен отрицательной бесконечности. Это записывается как: \[ \lim_{x \to a} f(x) = -\infty. \]
          • Пределы Примеры

            Используйте предельные законы для решения:

            \[ \lim_{x \to -3} (4x+2) \]

            Решение :

            Чтобы решить этот предел предельные законы по одному. Имейте в виду, что на каждом этапе вам нужно проверять, существует ли ограничение, прежде чем применять закон. Для применения закона должен существовать новый предел.

            1. Применить закон сумм.\[ \lim_{x \to -3} (4x+2) = \lim_{x \to -3} 4x + \lim_{x \to -3} 2 \]
            2. Применить постоянный кратный закон.\[ \lim_{x \to -3} (4x+2) = 4 \cdot \lim_{x \to -3} x + \lim_{x \to -3} 2 \ ]
            3. Применить базовый предел.\[ \lim_{x \to -3} (4x+2) = 4 \cdot (-3) + 2\]
            4. Упростить.\[ \lim_{x \to -3 } (4x+2) = -10\]

            Ограничения — ключевые выводы

            • Ограничения — это определение поведения функции при приближении к определенной точке или значению.
            • Математическое обозначение предела: \[ \lim_{x \to a} f(x) = L \]
            • Пределы можно оценить интуитивно, используя таблицу функциональных значений или график функции.
            • Существует несколько предельных законов, значительно упрощающих оценку пределов:
              • Два важных предела\[ \begin{align}1. \; & \lim_{x \to a} x = a \\2. \; & \lim_{x \to a} c = c\end{align} \]
              • Закон суммы для пределов:\[ \lim_{x \to a} (f(x) + g(x)) = \lim_ {x \to a} f(x) + \lim_{x \to a} g(x) = L + M \]
              • Разностный закон для пределов:\[ \lim_{x \to a} (f(x ) – g(x)) = \lim_{x \to a} f(x) – \lim_{x \to a} g(x) = L – M \] 9{n} \mbox{ для каждого положительного целого числа } n \]
              • Корневой закон для пределов:\[ \lim_{x \to a} \sqrt[n]{f(x)} = \sqrt[n]{\ lim_{x \to a} f(x)} = \sqrt[n]{L} \mbox{ для всех } L \mbox{, если } n \mbox{ нечетно, и для } L \geq 0 \mbox{ if } n \mbox{ равно} \]

            Часто задаваемые вопросы о пределах

            В математике пределы — это значения, к которым приближаются функции, когда их входные данные приближаются к некоторому значению. То, как вы можете думать о пределе, состоит в том, что вход функции становится все ближе и ближе к некоторому значению, функция становится все ближе и ближе и остается близкой к пределу.

            Пределы — это инструмент, который можно использовать для определения поведения функции в непосредственной близости от ее неопределенных точек или диапазонов точек. Пределы являются основой исчисления и используются для определения непрерывности, производных и интегралов.

            В базовом исчислении предел — это значение, к которому приближается функция, когда ее вход приближается к некоторому значению.

            Чтобы найти предел функции, вы напрямую подставляете значение, к которому приближается независимая переменная (обычно x), и решаете.

            Если это невозможно, вы можете попробовать некоторые алгебраические манипуляции с функцией, такие как

            • вынесение на множители общих членов,
            • умножение дроби на сопряженную,
            • использование тригонометрических преобразований,
            • просмотр графика функция для пределов в бесконечности,
            • или использование правила Лопиталя для неопределенных форм, таких как 0/0.

            После того как вы упростите предел с помощью этих методов, вы можете найти предел с помощью прямой подстановки.

            Проще говоря, предела не существует, если функциональные значения не приближаются к одному значению. Это происходит в следующих случаях:

            • функция резко колеблется при приближении к пределу
            • односторонние пределы не равны
            • когда предел равен бесконечности (слева, справа или с обеих сторон)

            Окончательный опрос о пределах

            Вопрос

            Каково интуитивное определение предела?

            Показать ответ

            Ответ

            Думайте о пределе функции с числом а как о действительном числе L, к которому функциональные значения приближаются по мере того, как его значения x приближаются к а, при условии, что число L существует.

            Показать вопрос

            Вопрос

            Каково формальное определение предела?

            Показать ответ

            Ответ

            Пусть f(x) — функция, определенная для всех значений открытого интервала, содержащего a (возможно, кроме a), и пусть L — действительное число. Если все значений f(x) приближаются к действительному числу L так же, как значения x, за исключением x = a, приближаются к числу a, то можно сказать, что предел f(x), когда x  приближается к  , равен L .

            Или, проще говоря:

            По мере того, как x становится все ближе и ближе к a, f(x) становится все ближе и ближе и остается близкой к L. Идея предела представлена ​​с использованием математической записи как:


            Показать вопрос

            Вопрос

            Каковы два способа интуитивного определения предела?

            Показать ответ

            Ответить

            Используя таблицу функциональных значений.

            Показать вопрос

            Вопрос

            Каковы два важных ограничения?

            Показать ответ

            Ответ


            Показать вопрос

            Вопрос

            Когда предел не существует?

            Показать ответ

            Ответ

            Предел не существует, если функциональные значения не приближаются к одному вещественному значению.

            Показать вопрос

            Вопрос

            В каких случаях предел не существует?

            Показать ответ

            Ответ

            Когда функция колеблется все сильнее и сильнее по мере приближения к пределу.

            Показать вопрос

            Вопрос

            Каковы 3 типа бесконечных пределов?

            Показать ответ

            Ответ

            1. Бесконечный предел слева
            2. Бесконечный предел справа
            3. Двусторонний бесконечный предел

            Показать вопрос

            Вопрос

            Каков закон сумм для пределов?

            Показать ответ

            Ответ

            Пусть f(x) и g(x) определены для всех  на открытом интервале, содержащем a. Предположим, что L и M — действительные числа, такие как:

            и

            Пусть c — константа. Тогда закон сумм для пределов:



            Показать вопрос

            Вопрос

            В чем разница закона для пределов?

            Показать ответ

            Ответ

            Пусть f(x) и g(x) определены для всего открытого интервала, содержащего a. Предположим, что L и M — действительные числа, такие как:

            и

            Пусть c — константа. Тогда разностный закон для пределов:


            Показать вопрос

            Вопрос

            Что такое постоянный кратный закон для пределов?

            Показать ответ

            Ответ

            Пусть f(x) и g(x) определены для всего открытого интервала, содержащего a. Предположим, что L и M — действительные числа, например:

            и

            Пусть c — константа. Тогда постоянный кратный закон для пределов:


            Показать вопрос

            Вопрос

            Каков закон произведения для пределов?

            Показать ответ

            Ответ

            Пусть f(x) и g(x) определены для всего открытого интервала, содержащего a. Предположим, что L и M — действительные числа, такие как:

            и

            Пусть c — константа. Тогда закон произведения для пределов:


            Показать вопрос

            Вопрос

            Каков закон частных для пределов?

            Показать ответ

            Ответ

            Пусть f(x) и g(x) определены для всего открытого интервала, содержащего a. Предположим, что L и M — действительные числа, такие как:

            и

            Пусть c — константа. Тогда частное для пределов:


            Показать вопрос

            Вопрос

            Что такое степенной закон для пределов?

            Показать ответ

            Ответ

            Пусть f(x) и g(x) определены для всего открытого интервала, содержащего a. Предположим, что L и M — действительные числа, такие как:

            и

            Пусть c — константа. Тогда степенной закон для пределов:


            Показать вопрос

            Вопрос

            Каков основной закон для пределов?

            Показать ответ

            Ответ

            Пусть f(x) и g(x) определены для всего открытого интервала, содержащего a. Предположим, что L и M — действительные числа, такие что:

            и

            Пусть c — константа. Тогда основной закон для пределов:


            Показать вопрос

            Подробнее о лимитах

            Откройте для себя подходящий контент для ваших тем

            Не нужно обманывать, если у вас есть все необходимое для успеха! Упаковано в одно приложение!

            Учебный план

            Будьте идеально подготовлены вовремя с индивидуальным планом.

            Тесты

            Проверьте свои знания с помощью игровых тестов.

            Карточки

            Создавайте и находите карточки в рекордно короткие сроки.

            Заметки

            Создавайте красивые заметки быстрее, чем когда-либо прежде.

            Учебные наборы

            Все учебные материалы в одном месте.

            Документы

            Загружайте неограниченное количество документов и сохраняйте их в Интернете.

            Study Analytics

            Определите сильные и слабые стороны вашего исследования.

            Еженедельные цели

            Ставьте индивидуальные учебные цели и зарабатывайте баллы за их достижение.

            Умные напоминания

            Хватит откладывать на потом наши напоминания об учебе.

            Награды

            Зарабатывайте очки, открывайте значки и повышайте уровень во время учебы.

            Волшебный маркер

            Создавайте карточки в заметках полностью автоматически.

            Интеллектуальное форматирование

            Создавайте самые красивые учебные материалы, используя наши шаблоны.

            2.2 Предел функции. Исчисление, том 1

            Цели обучения

            • 2.2.1 Используя правильные обозначения, опишите предел функции.
            • 2.2.2 Используйте таблицу значений, чтобы оценить предел функции или определить, когда предел не существует.
            • 2.2.3 Используйте график, чтобы оценить предел функции или определить, когда предел не существует.
            • 2.2.4 Дайте определение односторонним ограничениям и приведите примеры.
            • 2.2.5 Объясните связь между односторонними и двусторонними ограничениями.
            • 2.2.6 Используя правильные обозначения, опишите бесконечный предел.
            • 2.2.7 Задайте вертикальную асимптоту.

            Концепция предела или ограничивающего процесса, необходимая для понимания исчисления, существует уже тысячи лет. На самом деле ранние математики использовали ограничивающий процесс для получения все более и более точных приближений площадей кругов. Однако формальное определение предела — в том виде, в каком мы его знаем и понимаем сегодня — появилось только в конце XIX в. век. Поэтому мы начинаем наши поиски понимания пределов, как это делали наши предки-математики, используя интуитивный подход. В конце этой главы, вооружившись концептуальным пониманием пределов, мы рассмотрим формальное определение предела.

            Начнем исследование пределов с рассмотрения графиков функций

            f(x)=x2−4x−2,g(x)=|x−2|x−2, и h(x)= 1(x−2)2,f(x)=x2−4x−2,g(x)=|x−2|x−2, и h(x)=1(x−2)2,

            , которые показано на рисунке 2.12. В частности, давайте сосредоточим наше внимание на поведении каждого графика при x=2.x=2 и около него.

            2 и х = -1 для х

            Рисунок 2.12 Эти графики показывают поведение трех различных функций в районе x=2.x=2.

            Каждая из трех функций не определена при x=2,x=2, но если мы делаем это утверждение и никакое другое, мы даем очень неполную картину поведения каждой функции вблизи x=2.x=2 . Чтобы более полно выразить поведение каждого графа вблизи 2, нам нужно ввести понятие предела.

            Интуитивное определение предела

            Давайте сначала внимательно посмотрим, как функция f(x)=(x2−4)/(x−2)f(x)=(x2−4)/(x−2) ведет себя при x=2x=2. на рисунке 2.12. Поскольку значения x приближаются к 2 по обе стороны от 2, значения y=f(x)y=f(x) приближаются к 4. Математически мы говорим, что предел f(x)f(x) как x приближается к 2 равно 4. Символически мы выражаем этот предел как

            limx→2f(x)=4.limx→2f(x)=4.

            Из этого очень краткого неформального взгляда на один предел давайте начнем разрабатывать интуитивное определение предела. Мы можем думать о пределе функции в числе a как единственное действительное число L , к которому функциональные значения приближаются, поскольку x -значения приближаются к a, при условии, что такое действительное число L существует. Сформулировав более тщательно, мы имеем следующее определение:

            Определение

            Пусть f(x)f(x) — функция, определенная для всех значений в открытом интервале, содержащем a , за возможным исключением самого a , и пусть L — действительное число. Если все значений функции f(x)f(x) приближаются к действительному числу L , поскольку значения x(≠a)x(≠a) приближаются к числу a , то мы говорим, что предел f(x)f(x) при приближении x к a равно L . (Более кратко, поскольку x становится ближе к a , f(x)f(x) приближается и остается близким к L .) Символически мы выражаем эту идею как

            limx→af(x)= L.limx→af(x)=L.

            (2.3)

            Мы можем оценить пределы, составив таблицы функциональных значений и просмотрев их графики. Этот процесс описан в следующей стратегии решения проблем.

            Стратегия решения проблем

            Стратегия решения проблем: оценка предела с помощью таблицы функциональных значений
            1. Чтобы вычислить limx→af(x),limx→af(x), мы начнем с заполнения таблицы функциональных значений. Мы должны выбрать два набора значений x — один набор значений приближается к и меньше , а другой набор значений приближается к и больше . В таблице 2.1 показано, как могут выглядеть ваши таблицы.
              х ф(х)ф(х) х ф(х)ф(х)
              а-0,1а-0,1 f(a-0,1)f(a-0,1) а+0,1а+0,1 ф(а+0,1)ф(а+0,1)
              а-0,01а-0,01 f(a-0,01)f(a-0,01) а+0,01а+0,01 f(а+0,01)f(а+0,01)
              а-0,001а-0,001 f(a-0,001)f(a-0,001) а+0,001а+0,001 ф(а+0,001)ф(а+0,001)
              а-0,0001а-0,0001 f(a-0,0001)f(a-0,0001) а+0,0001а+0,0001 ф(а+0,0001)ф(а+0,0001)
              При необходимости используйте дополнительные значения. При необходимости используйте дополнительные значения.

              Стол 2.1 Таблица функциональных значений для limx→af(x)limx→af(x)

            2. Далее давайте посмотрим на значения в каждом из столбцов f(x)f(x) и определим, приближаются ли значения к одному значению по мере продвижения вниз по каждому столбцу. В наших столбцах мы рассматриваем последовательность f(a-0,1),f(a-0,01),f(a-0,001),f(a-0,0001),f(a-0,1),f(a-0,01). ),f(a-0,001).,f(a-0,0001) и т.д., и f(a+0,1),f(a+0,01),f(a+0,001),f(a+0,0001), f(a+0,1),f(a+0,01),f(a+0,001),f(a+0,0001) и так далее. ( Примечание : Хотя мы выбрали x -значения a±0,1,a±0,01,a±0,001,a±0,0001,a±0,1,a±0,01,a±0,001,a±0,0001 и т. д., и эти значения, вероятно, будут работать почти каждый раз, в очень редких случаях нам может потребоваться изменить наш выбор.)
            3. Если оба столбца приближаются к общему y -значению L , мы устанавливаем limx→af(x)=L. limx→af(x)=L. Мы можем использовать следующую стратегию для подтверждения результата, полученного из таблицы, или в качестве альтернативного метода оценки предела.
            4. С помощью графического калькулятора или компьютерной программы, которая позволяет нам графически отображать функции, мы можем построить функцию f(x),f(x), убедившись, что функциональные значения f(x)f(x) для x -значения близки и в нашем окне. Мы можем использовать функцию трассировки, чтобы перемещаться по графику функции и наблюдать за показаниями y , когда значения х приближаются к и . Если значения y приближаются к L , поскольку наши значения x приближаются к a в обоих направлениях, тогда limx→af(x)=L.limx→af(x)=L. Возможно, нам придется увеличить наш график и повторить этот процесс несколько раз.

            Мы применяем эту стратегию решения проблем для вычисления предела в примере 2.4.

            Пример 2,4

            Оценка предела с помощью таблицы функциональных значений 1

            Вычислить limx→0sinxxlimx→0sinxx с помощью таблицы функциональных значений.

            Решение

            Мы вычислили значения f(x)=(sinx)/xf(x)=(sinx)/x для значений x , указанные в таблице 2.2.

            х sinxxsinxx х sinxxsinxx
            −0,1 0,998334166468 0,1 0,998334166468
            −0,01 0,999983333417 0,01 0,999983333417
            −0,001 0,999999833333 0,001 0,999999833333
            −0,0001 0,999999998333 0,0001 0,999999998333

            Стол 2. 2 Таблица функциональных значений для limx→0sinxxlimx→0sinxx

            Примечание : Значения в этой таблице были получены с помощью калькулятора и с использованием всех мест, указанных в выходных данных калькулятора.

            Читая каждый столбец (sinx)x(sinx)x, мы видим, что значения в каждом столбце приближаются к единице. Таким образом, вполне разумно заключить, что limx→0sinxx=1.limx→0sinxx=1. График f(x)=(sinx)xf(x)=(sinx)x, построенный калькулятором или компьютером, будет похож на график, показанный на рис. 2.13, и он подтверждает нашу оценку.

            Рисунок 2.13 График f(x)=(sinx)/xf(x)=(sinx)/x подтверждает оценку из табл. 2.2.

            Пример 2,5

            Оценка предела с использованием таблицы функциональных значений 2

            Вычисление limx→4x-2x-4limx→4x-2x-4 с использованием таблицы функциональных значений.

            Решение

            Как и прежде, мы используем таблицу — в данном случае Таблицу 2. 3 — для перечисления значений функции для заданных значений x .

            х х-2х-4х-2х-4 х х-2х-4х-2х-4
            3,9 0,251582341869 4.1 0,248456731317
            3,99 0,25015644562 4.01 0,24984394501
            3,999 0,250015627 4. 001 0,249984377
            3,9999 0,250001563 4.0001 0,249998438
            3,99999 0,25000016 4.00001 0,24999984

            Стол 2.3 Таблица функциональных значений для limx→4x−2x−4limx→4x−2x−4

            Изучив эту таблицу, мы видим, что функциональные значения меньше 4 уменьшаются до 0,25, тогда как функциональные значения больше 4 кажутся увеличивается до 0,25. Делаем вывод, что limx→4x−2x−4=0,25.limx→4x−2x−4=0,25. Мы подтверждаем эту оценку, используя график f(x)=x−2x−4f(x)=x−2x−4, показанный на рис. 2.14.

            Рисунок 2.14 График f(x)=x−2x−4f(x)=x−2x−4 подтверждает оценку из табл. 2.3.

            Контрольно-пропускной пункт 2,4

            Оцените limx→11x-1x-1limx→11x-1x-1, используя таблицу функциональных значений. Используйте график, чтобы подтвердить свою оценку.

            На данный момент из примеров 2.4 и 2.5 мы видим, что может быть так же просто, если не проще, оценить предел функции, исследуя ее график, как и оценить предел, используя таблицу функциональных значений. . В примере 2.6 мы оцениваем предел исключительно по графику, а не по таблице функциональных значений.

            Пример 2,6

            Оценка предела с помощью графика

            Для g(x)g(x), показанного на рис. 2.15, вычислить limx→−1g(x).limx→−1g(x).

            Рисунок 2,15 График g(x)g(x) включает одно значение не на гладкой кривой.

            Решение

            Несмотря на то, что g(-1)=4,g(-1)=4, поскольку значения x приближаются к -1 с любой стороны, значения g(x)g(x) приближаются к 3. Следовательно, limx→−1g(x)=3.limx→−1g(x)=3. Заметим, что мы можем определить этот предел, даже не зная алгебраического выражения функции.

            На основании примера 2.6 делаем следующее наблюдение: возможно, что предел функции существует в точке и функция может быть определена в этой точке, но предел функции и значение функции момент может быть другим.

            Контрольно-пропускной пункт 2,5

            Используйте график h(x)h(x) на рис. 2.16, чтобы вычислить limx→2h(x),limx→2h(x), если это возможно.

            Рисунок 2.16

            Глядя на таблицу функциональных значений или глядя на график функции, мы получаем полезную информацию о значении предела функции в данной точке. Однако эти методы слишком полагаются на догадки. Со временем нам потребуется разработать альтернативные методы оценки пределов. Эти новые методы носят более алгебраический характер, и мы рассмотрим их в следующем разделе; однако на этом этапе мы вводим два специальных ограничения, которые лежат в основе будущих методов.

            Теорема 2.1

            Два важных ограничения

            Пусть a — действительное число, а c — константа.

            1. limx→ax=alimx→ax=a

              (2.4)

            2. limx→ac=climx→ac=c

              (2.5)

            Мы можем сделать следующие замечания относительно этих двух пределов.

            1. Для первого предела обратите внимание, что когда x приближается к a , то же самое делает f(x),f(x), потому что f(x)=x.f(x)=x. Следовательно, limx→ax=a.limx→ax=a.
            2. Для второго предела см. Таблицу 2.4.
            х f(x)=cf(x)=c х f(x)=cf(x)=c
            а-0,1а-0,1 с а+0,1а+0,1 с
            а-0,01а-0,01 с а+0,01а+0,01 с
            а-0,001а-0,001 с а+0,001а+0,001 с
            а-0,0001а-0,0001 с а+0,0001а+0,0001 с

            Стол 2,4 Таблица функциональных значений для limx→ac=climx→ac=c

            Обратите внимание, что для всех значений x (независимо от того, приближаются ли они к a ), значения f(x)f(x) остаются постоянными на уровне с . У нас нет другого выбора, кроме как заключить limx→ac=c.limx→ac=c.

            Существование предела

            Когда мы рассматриваем предел в следующем примере, имейте в виду, что для того, чтобы предел функции существовал в точке, функциональные значения должны приближаться к единственному вещественному значению в этой точке. Если функциональные значения не приближаются к единому значению, то предела не существует.

            Пример 2,7

            Оценка несуществующего предела

            Вычислить limx→0sin(1/x)limx→0sin(1/x), используя таблицу значений.

            Решение

            В таблице 2.5 перечислены значения функции sin(1/x)sin(1/x) для заданных значений x .

            187187
            х грех (1x) грех (1x) х грех (1x) грех (1x)
            −0,1 0,544021110889 0,1 −0,544021110889
            −0,01 0,50636564111 0,01 −0,50636564111
            −0,001 −0,8268795405312 0,001 0,826879540532
            −0,0001 0,305614388888 0,0001 −0,305614388888
            −0,00001 −0,035748797987 0,00001 0,035748797987
            −0,000001 0,3499 0,000001 −0,3499

            Стол 2,5 Таблица функциональных значений для limx→0sin(1x)limx→0sin(1x)

            Изучив таблицу функциональных значений, мы видим, что значения y не приближаются ни к одному единственному значению. Получается, что предела не существует. Прежде чем сделать такой вывод, давайте подойдем более системно. Возьмите следующую последовательность x -значений, приближающихся к 0:

            2π,23π,25π,27π,29π,211π,….2π,23π,25π,27π,29π,211π,….

            Соответствующие y -значения равны

            1,−1,1,−1,1,−1,….1,−1,1,−1,1,−1,….

            На данный момент мы действительно можем заключить, что limx→0sin(1/x)limx→0sin(1/x) не существует. (Математики часто сокращают «не существует» до DNE. Таким образом, мы будем писать limx→0sin(1/x)limx→0sin(1/x) DNE.) График f(x)=sin(1/x) f(x)=sin(1/x) показано на рис. 2.17, и оно дает более четкое представление о поведении sin(1/x)sin(1/x) при приближении x к 0. Вы можете видеть, что sin (1/x)sin(1/x) колеблется все более дико между -1 и 1 как x приближается к 0.

            Рисунок 2.17 График f(x)=sin(1/x)f(x)=sin(1/x) быстро колеблется между −1 и 1, когда x приближается к 0,

            .

            Контрольно-пропускной пункт 2,6

            Используйте таблицу функциональных значений для вычисления limx→2|x2−4|x−2, limx→2|x2−4|x−2, если это возможно.

            Односторонние пределы

            Иногда указание на то, что предел функции не существует в какой-то точке, не дает нам достаточно информации о поведении функции в этой конкретной точке. Чтобы убедиться в этом, вернемся к функции g(x)=|x−2|/(x−2)g(x)=|x−2|/(x−2), введенной в начале раздела (см. Рисунок 2.12(б)). Когда мы выбираем значения x близко к 2, g(x)g(x) не приближается ни к одному значению, поэтому предела, когда x приближается к 2, не существует, то есть limx→2g(x)limx→2g(x) ДНЭ. Однако само по себе это утверждение не дает нам полной картины поведения функции в районе x -значения 2. Для более точного описания введем понятие одностороннего предела. Для всех значений слева от 2 (или – отрицательная сторона 2) g(x)=-1. g(x)=-1. Таким образом, когда x приближается к 2 слева, g(x)g(x) приближается к -1. Математически мы говорим, что предел как x приближается к 2 слева равно −1. Символически мы выражаем эту идею как

            limx→2−g(x)=−1.limx→2−g(x)=−1.

            Точно так же, когда x приближается к 2 справа (или с положительной стороны ), g(x)g(x) приближается к 1. Символически мы выражаем эту идею как

            limx→2+g(x)=1.limx→2+g(x)=1.

            Теперь мы можем дать неформальное определение односторонних пределов.

            Определение

            Определим два типа односторонних пределов .

            Предел слева: Пусть f(x)f(x) будет функцией, определенной для всех значений в открытом интервале формы ( c , a ), и пусть L будет действительным количество. Если значения функции f(x)f(x) приближаются к действительному числу L , как значения x (где x a , то мы говорим, что L — это предел f(x)f(x), поскольку x приближается к a слева. Символически мы выражаем эту идею как

            limx→a−f(x)=L.limx→a−f(x)=L.

            (2.6)

            Предел справа: Пусть f(x)f(x) функция, определенная при всех значениях в открытом интервале вида (a,c),(a,c), и пусть L будет действительным числом. Если значения функции f(x)f(x) приближаются к действительному числу L, как значения x (где x>a)x>a) приближаются к числу a , то мы говорим, что L является пределом f(x)f(x), поскольку x приближается к a справа. Символически мы выражаем эту идею как

            limx→a+f(x)=L.limx→a+f(x)=L.

            (2,7)

            Пример 2,8

            Оценка односторонних пределов

            Для функции f(x)={x+1ifx<2x2−4ifx≥2,f(x)={x+1ifx<2x2−4ifx≥2 оцените каждый из следующих пределов .

            1. limx→2−f(x)limx→2−f(x)
            2. limx→2+f(x)limx→2+f(x)
            Решение

            Можно снова использовать таблицы функциональных значений Таблица 2.6. Обратите внимание, что для значений x меньше 2, мы используем f(x)=x+1f(x)=x+1, а для значений x больше 2 мы используем f(x)=x2−4.f(x) =x2−4.

            х f(x)=x+1f(x)=x+1 х f(x)=x2−4f(x)=x2−4
            1,9 2,9 2. 1 0,41
            1,99 2,99 2,01 0,0401
            1.999 2,999 2.001 0,004001
            1,9999 2,9999 2.0001 0,00040001
            1,99999 2,99999 2.00001 0,0000400001

            Стол 2,6 Таблица функциональных значений для f(x)={x+1ifx<2x2−4ifx≥2f(x)={x+1ifx<2x2−4ifx≥2

            На основании этой таблицы можно заключить, что a. limx→2−f(x)=3limx→2−f(x)=3 и б. limx→2+f(x)=0.limx→2+f(x)=0. Следовательно, (двусторонний) предел f(x)f(x) не существует при x=2. x=2. На рис. 2.18 показан график f(x)f(x), который подтверждает наш вывод об этих пределы.

            = 2. Первая часть представляет собой линию с точкой пересечения x в точке (-1, 0) и точкой пересечения y в точке (0,1). В точке (2,3) есть незакрашенный кружок, где должна быть конечная точка. Второй кусок — правая половина параболы, раскрывающейся вверх. Вершина в точке (2,0) представляет собой сплошной круг.”>

            Рисунок 2.18 График функции f(x)={x+1ifx<2x2−4ifx≥2f(x)={x+1ifx<2x2−4ifx≥2 имеет разрыв в точке x=2.x=2.

            Контрольно-пропускной пункт 2,7

            Используйте таблицу функциональных значений для оценки следующих пределов, если это возможно.

            1. limx→2−|x2−4|x−2limx→2−|x2−4|x−2
            2. limx→2+|x2−4|x−2limx→2+|x2−4|x−2

            Рассмотрим теперь связь между пределом функции в точке и пределами справа и слева в этой точке. Кажется очевидным, что если предел справа и предел слева имеют общее значение, то это общее значение является пределом функции в этой точке. Точно так же, если предел слева и предел справа принимают разные значения, предел функции не существует. Эти выводы резюмируются в разделе «Связь односторонних и двусторонних пределов».

            Теорема 2.2

            Связь односторонних и двусторонних пределов

            Пусть f(x)f(x) — функция, определенная для всех значений в открытом интервале, содержащем a , за возможным исключением самого a , и пусть L — действительное число. Тогда

            limx→af(x)=L.если и только еслиlimx→a−f(x)=Landlimx→a+f(x)=L.limx→af(x)=L.если и только еслиlimx→ a−f(x)=Landlimx→a+f(x)=L.

            Бесконечные пределы

            Оценка предела функции в точке или оценка предела функции справа и слева в точке помогает нам охарактеризовать поведение функции при заданном значении. Как мы увидим, мы можем также описать поведение функций, не имеющих конечных пределов.

            Теперь обратим внимание на h(x)=1/(x−2)2,h(x)=1/(x−2)2, третью и последнюю функцию, введенную в начале этого раздела (см. рис. 2.12(с)). Из его графика видно, что по мере того, как значения x приближаются к 2, значения h(x)=1/(x−2)2h(x)=1/(x−2)2 становятся все больше и больше, и, на самом деле, стать бесконечным. Математически мы говорим, что предел h(x)h(x) при приближении x к 2 равен положительной бесконечности. Символически мы выражаем эту идею как

            limx→2h(x)=+∞.limx→2h(x)=+∞.

            В более общем смысле мы определяем бесконечные пределы следующим образом:

            Определение

            Мы определяем три типа бесконечных пределов .

            Бесконечные пределы слева: Пусть f(x)f(x) — функция, определенная для всех значений в открытом интервале формы (b,a).(b,a).

            1. Если значения f(x)f(x) неограниченно возрастают по мере того, как значения x (где x a , то мы говорим, что предел равен x приближается к а слева является положительной бесконечностью, и мы пишем

              limx→a−f(x)=+∞. limx→a−f(x)=+∞.

              (2.8)

            2. Если значения f(x)f(x) неограниченно уменьшаются по мере того, как значения x (где x a , то мы говорим, что предел равен x приближается к a слева — это отрицательная бесконечность, и мы пишем

              limx→a−f(x)=−∞.limx→a−f(x)=−∞.

              (2,9)

            Бесконечные пределы справа : Пусть f(x)f(x) — функция, определенная для всех значений в открытом интервале формы (a,c).(a,c).

            1. Если значения f(x)f(x) неограниченно возрастают по мере того, как значения x (где x>a)x>a) приближаются к числу a , то мы говорим, что предел равен x приближается к a справа является положительной бесконечностью, и мы пишем

              limx→a+f(x)=+∞. limx→a+f(x)=+∞.

              (2.10)

            2. Если значения f(x)f(x) неограниченно уменьшаются по мере того, как значения x (где x>a)x>a) приближаются к числу a , то мы говорим, что предел равен x приближается к a справа — это отрицательная бесконечность, и мы пишем

              limx→a+f(x)=−∞.limx→a+f(x)=−∞.

              (2.11)

            Двусторонний бесконечный предел: Пусть f(x)f(x) определено для всех x≠ax≠a в открытом интервале, содержащем a .

            1. Если значения f(x)f(x) неограниченно возрастают по мере того, как значения x (где x≠a)x≠a) приближаются к числу a , то мы говорим, что предел равен x приближается к a есть положительная бесконечность, и мы пишем

              limx→af(x)=+∞. limx→af(x)=+∞.

              (2.12)

            2. Если значения f(x)f(x) неограниченно уменьшаются по мере того, как значения x (где x≠a)x≠a) приближаются к числу a , то говорят, что предел равен x приближается к , а — это отрицательная бесконечность, и мы пишем

              limx→af(x)=−∞.limx→af(x)=−∞.

              (2.13)

            Важно понимать, что когда мы пишем операторы типа limx→af(x)=+∞limx→af(x)=+∞ или limx→af(x)=−∞limx→af(x)=−∞ мы описываем поведение функции, как только что определили ее. Мы не утверждаем, что предел существует. Чтобы предел функции f(x)f(x) существовал на уровне a , он должен приближаться к действительному числу 9.1397 L , поскольку x приближается к a . Тем не менее, если, например, limx→af(x)=+∞, limx→af(x)=+∞, мы всегда будем писать limx→af(x)=+∞limx→af(x)=+∞, а не чем limx→af(x)limx→af(x) ДНЭ.

            Пример 2,9

            Распознавание бесконечного предела

            Оцените каждый из следующих пределов, если это возможно. Используйте таблицу функциональных значений и график f(x)=1/xf(x)=1/x, чтобы подтвердить свой вывод.

            1. limx→0−1xlimx→0−1x
            2. limx→0+1xlimx→0+1x
            3. limx→01xlimx→01x
            Решение

            Начните с построения таблицы функциональных значений.

            х 1x1x х 1x1x
            −0,1 −10 0,1 10
            −0,01 −100 0,01 100
            −0,001 −1000 0,001 1000
            −0,0001 −10 000 0,0001 10 000
            −0,00001 −100 000 0,00001 100 000
            −0,000001 −1 000 000 0,000001 1 000 000

            Стол 2,7 Таблица функциональных значений для f(x)=1xf(x)=1x

            1. Значения 1/x1/x неограниченно уменьшаются как x приближается к 0 слева. Мы заключаем, что

              limx→0−1x=−∞.limx→0−1x=−∞.

            2. Значения 1/x1/x неограниченно возрастают по мере приближения x к 0 справа. Мы заключаем, что

              limx→0+1x=+∞.limx→0+1x=+∞.

            3. Поскольку limx→0−1x=−∞limx→0−1x=−∞ и limx→0+1x=+∞limx→0+1x=+∞ имеют разные значения, мы заключаем, что

              limx→01xDNE.limx→01xDNE .

            График f(x)=1/xf(x)=1/x на рис. 2.19 подтверждает эти выводы.

            Рисунок 2.19 График f(x)=1/xf(x)=1/x подтверждает, что предел, когда x приближается к 0, не существует.

            Контрольно-пропускной пункт 2,8

            Оцените каждый из следующих пределов, если это возможно. Используйте таблицу функциональных значений и график f(x)=1/x2f(x)=1/x2, чтобы подтвердить свой вывод.

            1. limx→0−1x2limx→0−1×2
            2. limx→0+1x2limx→0+1×2
            3. лимкс→01×2лимкс→01×2

            Полезно отметить, что функции вида f(x)=1/(x−a)n,f(x)=1/(x−a)n, где n — положительное целое число, имеет бесконечные пределы, поскольку x приближается к a либо слева, либо справа (рис. 2.20). Эти ограничения суммированы в разделе «Бесконечные пределы от положительных целых чисел».

            Рисунок 2.20 Функция f(x)=1/(x−a)nf(x)=1/(x−a)n имеет бесконечные пределы от до .

            Теорема 2.3

            Бесконечные пределы от положительных целых чисел

            Если n — положительное четное целое число, то

            limx→a1(x−a)n=+∞.limx→a1(x−a)n=+∞.

            Если n — натуральное нечетное число, то

            limx→a+1(x−a)n=+∞limx→a+1(x−a)n=+∞

            и

            limx→ a−1(x−a)n=−∞.limx→a−1(x−a)n=−∞.

            Следует также отметить, что на графиках f(x)=1/(x−a)n,f(x)=1/(x−a)n точки на графике, имеющие x -координат, очень рядом с a очень близки к вертикальной линии x=a.x=a. То есть, когда x приближается к a , точки на графике f(x)f(x) ближе к линии x=a. x=a. Линия x=ax=a называется вертикальной асимптотой графика. Формально определим вертикальную асимптоту следующим образом:

            Определение

            Пусть f(x)f(x) — функция. Если выполняется какое-либо из следующих условий, то линия x=ax=a является вертикальной асимптотой функции f(x).f(x).

            limx→a−f(x)=+∞or−∞limx→a+f(x)=+∞or−∞orlimx→af(x)=+∞or−∞limx→a−f(x) =+∞или-∞limx→a+f(x)=+∞или-∞илиlimx→af(x)=+∞или-∞

            Пример 2.10

            Нахождение вертикальной асимптоты

            Оцените каждый из следующих пределов, используя Бесконечные пределы из положительных целых чисел. Определите любые вертикальные асимптоты функции f(x)=1/(x+3)4.f(x)=1/(x+3)4.

            1. limx→−3−1(x+3)4limx→−3−1(x+3)4
            2. limx→−3+1(x+3)4limx→−3+1(x+3)4
            3. limx→−31(x+3)4limx→−31(x+3)4
            Решение

            Мы можем напрямую использовать бесконечные пределы из положительных целых чисел.

            1. limx→−3−1(x+3)4=+∞limx→−3−1(x+3)4=+∞
            2. limx→−3+1(x+3)4=+∞limx→−3+1(x+3)4=+∞
            3. limx→−31(x+3)4=+∞limx→−31(x+3)4=+∞

            Функция f(x)=1/(x+3)4f(x)=1/(x+3)4 имеет вертикальную асимптоту x=−3.x=−3.

            Контрольно-пропускной пункт 2,9

            Оцените каждый из следующих пределов. Определите любые вертикальные асимптоты функции f(x)=1(x−2)3.f(x)=1(x−2)3.

            1. limx→2−1(x−2)3limx→2−1(x−2)3
            2. limx→2+1(x−2)3limx→2+1(x−2)3
            3. limx→21(x−2)3limx→21(x−2)3

            В следующем примере мы используем наши знания о различных типах ограничений для анализа поведения функции в нескольких различных точках.

            Пример 2.11

            Поведение функции в различных точках

            Используйте график f(x)f(x) на рис. 2.21 для определения каждого из следующих значений:

            1. limx→−4−f(x);limx→− 4+f(x);limx→−4f(x);f(−4)limx→−4−f(x);limx→−4+f(x);limx→−4f(x);f( −4)
            2. limx→−2−f(x);limx→−2+f(x);limx→−2f(x);f(−2)limx→−2−f(x);limx→−2+f (х);limx→−2f(x);f(−2)
            3. limx→1−f(x);limx→1+f(x);limx→1f(x);f(1)limx→1−f(x);limx→1+f(x);limx→ 1f(x);f(1)
            4. limx→3−f(x);limx→3+f(x);limx→3f(x);f(3)limx→3−f(x);limx→3+f(x);limx→ 3f(x);f(3)

            Рисунок 2. 21 График показывает f(x).f(x).

            Решение

            Используя бесконечные пределы из положительных целых чисел и график для справки, мы получаем следующие значения:

            1. limx→−4−f(x)=0;limx→−4+f(x)=0;limx→− 4f(x)=0;f(−4)=0limx→−4−f(x)=0;limx→−4+f(x)=0;limx→−4f(x)=0;f(− 4)=0
            2. limx→−2−f(x)=3.;limx→−2+f(x)=3;limx→−2f(x)=3;f(−2)limx→−2−f(x) =3.;limx→−2+f(x)=3;limx→−2f(x)=3;f(−2) не определено
            3. limx→1−f(x)=6;limx→1+f(x)=3;limx→1f(x)limx→1−f(x)=6;limx→1+f(x)=3 ;limx→1f(x) ДНЭ; f(1)=6f(1)=6
            4. limx→3−f(x)=−∞;limx→3+f(x)=−∞;limx→3f(x)=−∞;f(3)limx→3−f(x)=−∞ ;limx→3+f(x)=−∞;limx→3f(x)=−∞;f(3) не определено

            Контрольно-пропускной пункт 2.10

            Вычисление limx→1f(x)limx→1f(x) для показанного здесь f(x)f(x):

            Пример 2.12

            Начало главы: уравнение Эйнштейна

            Рисунок 2,22 (кредит: НАСА)

            В начале главы мы кратко упомянули, как Альберт Эйнштейн показал, что существует предел скорости движения любого объекта. Учитывая уравнение Эйнштейна для массы движущегося объекта, каково значение этой границы?

            Решение

            Нашей отправной точкой является уравнение Эйнштейна для массы движущегося объекта,

            m=m01−v2c2,m=m01−v2c2,

            , где m0m0 — масса объекта в состоянии покоя, v — его скорость, а c это скорость света. Чтобы увидеть, как масса изменяется при высоких скоростях, мы можем построить график отношения масс m/m0m/m0 как функцию отношения скоростей v/cv/c (рис. 2.23).

            Рисунок 2,23 На этом графике показано отношение масс как функция отношения скоростей в уравнении Эйнштейна для массы движущегося объекта.

            Мы можем видеть, что по мере того, как отношение скоростей приближается к 1, то есть когда скорость объекта приближается к скорости света, отношение масс неограниченно возрастает. Другими словами, функция имеет вертикальную асимптоту при v/c=1. v/c=1. Мы можем попробовать несколько значений этого отношения, чтобы проверить эту идею.

            вквк 1-v2c21-v2c2 мм0мм0
            0,99 0,1411 7.089
            0,999 0,0447 22,37
            0,9999 0,0141 70,71

            Стол 2,8 Отношение масс и скоростей движущегося объекта

            Таким образом, согласно таблице 2.8, если объект массой 100 кг движется со скоростью 0,9999 c , его масса становится равной 7071 кг. Поскольку ни один объект не может иметь бесконечную массу, мы заключаем, что ни один объект не может двигаться со скоростью, превышающей скорость света.

            Раздел 2.2 Упражнения

            В следующих упражнениях рассмотрим функцию f(x)=x2−1|x−1|.f(x)=x2−1|x−1|.

            30.

            [T] Заполните следующую таблицу для функции. Округлите ваши решения до четырех знаков после запятой.

            х ф(х)ф(х) х ф(х)ф(х)
            0,9 а. 1,1 эл.
            0,99 б. 1,01 ф.
            0,999 с. 1.001 г.
            0,9999 д. 1.0001 час.

            31.

            Что ваши результаты в предыдущем упражнении говорят о двустороннем пределе limx→1f(x)?limx→1f(x)? Объясните свой ответ.

            В следующих упражнениях рассмотрим функцию f(x)=(1+x)1/x.f(x)=(1+x)1/x.

            32.

            [T] Составьте таблицу, показывающую значения f для x=-0,01,-0,001,-0,0001,-0,00001x=-0,01,-0,001,-0,0001,-0,00001 и для x=0,01, 0,001,0,0001,0,00001.x=0,01,0,001,0,0001,0,00001. Округлите решения до пяти знаков после запятой.

            х ф(х)ф(х) х ф(х)ф(х)
            −0,01 а. 0,01 эл.
            −0,001 б. 0,001 ф.
            −0,0001 с. 0,0001 г.
            −0,00001 д. 0,00001 час.

            33.

            Что показывает таблица значений в предыдущем упражнении относительно функции f(x)=(1+x)1/x?f(x)=(1+x)1/x?

            34.

            К какой математической константе приближается предел в предыдущем упражнении?

            В следующих упражнениях используйте заданные значения для создания таблицы для оценки пределов. Округлите ваши решения до восьми знаков после запятой.

            35.

            [T] limx→0sin2xx;±0.1,±0.01,±0.001,±.0001limx→0sin2xx;±0.1,±0.01,±0.001,±.0001

            x sin2xxsin2xx х sin2xxsin2xx
            −0,1 а. 0,1 эл.
            −0,01 б. 0,01 ф.
            −0,001 с. 0,001 г.
            −0,0001 д. 0,0001 час.

            36.

            [T] limx→0sin3xxlimx→0sin3xx ±0,1, ±0,01, ±0,001, ±0,0001

            X sin3xxsin3xx х sin3xxsin3xx
            −0,1 а. 0,1 эл.
            −0,01 б. 0,01 ф.
            −0,001 с. 0,001 г.
            −0,0001 д. 0,0001 час.

            37.

            Используйте два предыдущих упражнения, чтобы предположить (угадать) значение следующего предела: limx→0sinaxxlimx→0sinaxx для a , положительное действительное значение.

            [T] В следующих упражнениях создайте таблицу значений, чтобы найти указанный предел. Округлите до восьми цифр.

            38.

            limx→2×2−4×2+x−6limx→2×2−4×2+x−6

            х х2-4х2+х-6х2-4х2+х-6 х х2-4х2+х-6х2-4х2+х-6
            1,9 а. 2.1 эл.
            1,99 б. 2,01 ф.
            1,999 с. 2,001 г.
            1,9999 д. 2.0001 час.

            39.

            limx→1(1−2x)limx→1(1−2x)

            x 1-2×1-2x х 1-2×1-2x
            0,9 а. 1,1 эл.
            0,99 б. 1,01 ф.
            0,999 с. 1.001 г.
            0,9999 д. 1.0001 час.

            40.

            limx→051−e1/xlimx→051−e1/x

            x 51-e1/x51-e1/x х 51-e1/x51-e1/x
            −0,1 г. н.э. 0,1 эл.
            −0,01 б. 0,01 ф.
            −0,001 с. 0,001 г.
            −0,0001 д. 0,0001 час.

            41.

            limz→0z−1z2(z+3)limz→0z−1z2(z+3)

            z z−1z2(z+3)z−1z2(z+3) с z−1z2(z+3)z−1z2(z+3)
            −0,1 а. 0,1 эл.
            −0,01 б. 0,01 ф.
            −0,001 с. 0,001 г.
            −0,0001 д. 0,0001 час.

            42.

            limt→0+costlimt→0+cost

            t стоимость стоимость
            0,1 а.
            0,01 б.
            0,001 с.
            0,0001 д.

            43.

            limx→21−2xx2−4limx→21−2xx2−4

            x 1-2xx2-41-2xx2-4 х 1-2xx2-41-2xx2-4
            1,9 а. 2.1 эл.
            1,99 б. 2,01 ф.
            1,999 с. 2.001 г.
            1,9999 д. 2.0001 час.

            [Т] В следующих упражнениях составьте таблицу значений и округлите ее до восьми значащих цифр. На основании таблицы значений сделайте предположение о пределе. Затем используйте калькулятор, чтобы построить график функции и определить предел. Верна ли была догадка? Если нет, то почему метод таблиц не работает?

            44.

            limθ→0sin(πθ)limθ→0sin(πθ)

            θ грех (πθ) грех (πθ) θ грех (πθ) грех (πθ)
            −0,1 а. 0,1 эл.
            −0,01 б. 0,01 ф.
            −0,001 с. 0,001 г.
            −0,0001 д. 0,0001 час.

            45.

            limα→0+1αcos(πα)limα→0+1αcos(πα)

            и 1αcos(πα)1αcos(πα)
            0,1 а.
            0,01 б.
            0,001 с.
            0,0001 д.

            В следующих упражнениях рассмотрите показанный здесь график функции y=f(x)y=f(x). Какие из утверждений о y=f(x)y=f(x) верны, а какие нет? Объясните, почему утверждение ложно.

            46.

            limx→10f(x)=0limx→10f(x)=0

            47.

            limx→−2+f(x)=3limx→−2+f(x)=3

            48.

            limx→−8f(x)=f(−8)limx→−8f(x)=f(−8)

            49.

            limx→6f(x)=5limx→6f(x)=5

            В следующих упражнениях используйте следующий график функции y=f(x)y=f(x), чтобы найти значения, если это возможно. Оцените, когда это необходимо.

            1. Первый отрезок линейный с наклоном 1 и проходит через начало координат. Его конечная точка представляет собой замкнутый круг в точке (1,1). Второй сегмент также является линейным с наклоном -1. Он начинается с незакрашенного круга в (1,2).”>

            50.

            limx→1−f(x)limx→1−f(x)

            51.

            limx→1+f(x)limx→1+f(x)

            52.

            limx→1f(x)limx→1f(x)

            53.

            limx→2f(x)limx→2f(x)

            54.

            ф(1)ф(1)

            В следующих упражнениях используйте показанный здесь график функции y=f(x)y=f(x), чтобы найти значения, если это возможно. Оцените, когда это необходимо.

            55.

            limx→0−f(x)limx→0−f(x)

            56.

            limx→0+f(x)limx→0+f(x)

            57.

            limx→0f(x)limx→0f(x)

            58.

            limx→2f(x)limx→2f(x)

            В следующих упражнениях используйте показанный здесь график функции y=f(x)y=f(x), чтобы найти значения, если это возможно. Оцените, когда это необходимо.

            2, имеет наклон 1 и начинается в незамкнутой окружности (2,2).”>

            59.

            limx→−2−f(x)limx→−2−f(x)

            60.

            limx→−2+f(x)limx→−2+f(x)

            61.

            limx→−2f(x)limx→−2f(x)

            62.

            limx→2−f(x)limx→2−f(x)

            63.

            limx→2+f(x)limx→2+f(x)

            64.

            limx→2f(x)limx→2f(x)

            В следующих упражнениях используйте показанный здесь график функции y=g(x)y=g(x), чтобы найти значения, если это возможно. Оцените, когда это необходимо.

            =0 и является левой половиной раскрывающейся вверх параболы с вершиной в замкнутом круге (0,3). Второй существует при x>0 и представляет собой правую половину раскрывающейся вниз параболы с вершиной в незамкнутой окружности (0,0).”>

            65.

            limx→0−g(x)limx→0−g(x)

            66.

            limx→0+g(x)limx→0+g(x)

            67.

            limx→0g(x)limx→0g(x)

            В следующих упражнениях используйте показанный здесь график функции y=h(x)y=h(x), чтобы найти значения, если это возможно. Оцените, когда это необходимо.

            68.

            limx→0−h(x)limx→0−h(x)

            69.

            limx→0+h(x)limx→0+h(x)

            70.

            limx→0h(x)limx→0h(x)

            В следующих упражнениях используйте показанный здесь график функции y=f(x)y=f(x), чтобы найти значения, если это возможно. Оцените, когда это необходимо.

            0, а в начале координат есть замкнутый круг.”>

            71.

            limx→0−f(x)limx→0−f(x)

            72.

            limx→0+f(x)limx→0+f(x)

            73.

            limx→0f(x)limx→0f(x)

            74.

            limx→1f(x)limx→1f(x)

            75.

            limx→2f(x)limx→2f(x)

            В следующих упражнениях нарисуйте график функции с заданными свойствами.

            76.

            limx→2f(x)=1,limx→4−f(x)=3,limx→4+f(x)=6,f(4)limx→2f(x)=1,limx→4− f(x)=3,limx→4+f(x)=6,f(4) не определено.

            77.

            As x→−∞ , f(x)→0,limx→-1−f(x)=-∞,As x→-∞ , f(x)→0,limx→-1−f(x) =−∞,limx→−1+f(x)=∞,limx→0f(x)=f(0),f(0)=1, при x→∞, f(x)→−∞limx→− 1+f(x)=∞,limx→0f(x)=f(0),f(0)=1, при x→∞, f(x)→−∞

            78.

            Как x→−∞, f(x)→2,limx→3−f(x)=-∞,As x→-∞, f(x)→2,limx→3-f(x)=- ∞,limx→3+f(x)=∞, При x→∞, f(x)→2,f(0)=−13limx→3+f(x)=∞, При x→∞, f(x )→2,f(0)=−13

            79.

            При x→−∞, f(x)→2,limx→−2f(x)=−∞,As x→−∞, f(x)→2,limx→−2f(x)=−∞, При x→∞, f(x)→2,f(0)=0При x→∞, f(x)→2,f(0)=0

            80.

            При x→−∞, f(x)→0,limx→−1−f(x)=∞,limx→−1+f(x)=−∞, As x→−∞, f(x) →0,limx→−1−f(x)=∞,limx→−1+f(x)=−∞,f(0)=−1,limx→1−f(x)=−∞,limx→ 1+f(x)=∞, As x→∞, f(x)→0f(0)=−1,limx→1−f(x)=−∞,limx→1+f(x)=∞, Поскольку x→∞, f(x)→0

            81.

            Ударные волны возникают во многих физических приложениях, от сверхновых до детонационных волн. График зависимости плотности ударной волны от расстояния, x , показано здесь. В основном нас интересует расположение фронта ударной волны, обозначенное на диаграмме xSFxSF.

            p2 и xsf по оси x. Он состоит из y= p1 от 0 до xsf, x = xsf от y= p1 до y=p2 и y=p2 для значений, больших или равных xsf.”>

            1. Вычислить limx→xSF+ρ(x).limx→xSF+ρ(x).
            2. Вычислить limx→xSF-ρ(x). limx→xSF-ρ(x).
            3. Вычислить limx→xSFρ(x).limx→xSFρ(x). Объясните физический смысл ваших ответов.

            82.

            Тренер по легкой атлетике использует камеру с быстрым затвором, чтобы оценить положение бегуна во времени. Здесь приведена таблица значений положения спортсмена в зависимости от времени, где х — положение бегуна в метрах, а t — время в секундах. Что такое limt→2x(t)?limt→2x(t)? Что это значит физически?

            т (сек) x (м)
            1,75 4,5
            1,95 6,1
            1,99 6,42
            2,01 6,58
            2,05 6,9
            2,25 8,5

            Исчисление

            .

            Какой простой пример ограничения в реальном мире?

            Спросил

            Модифицированный 2 года, 8 месяцев назад

            Просмотрено 142k раз

            $\begingroup$

            Сегодня утром я прочитал неофициальное определение лимита в Википедии:

            .

            Неформально функция f назначает выход $f(x)$ каждому входу $x$. функция имеет предел $L$ на входе $p$, если $f(x)$ “близка” к $L$ всякий раз, когда $x$ “близок” к $p$. Другими словами, $f(x)$ становится все ближе и ближе к $L$ по мере того, как $x$ приближается к $p$.

            Для меня это звучит как что-то, что можно было бы лучше описать как «цель».

            Если я возьму простую функцию, скажем, ту, которая умножает ввод только на $2$; и если мой лимит составляет $10$ при вводе $5$: тогда я описал что-то, что, кажется, соответствует элементам, содержащимся в определении Википедии. Я не верю, что это правильно. Для меня это похоже на задачу элементарной алгебры ($2p = 10$). Чтобы сделать это более математическим, я мог бы построить график вывода функции, когда я использую входные данные, отличные от $p$, но на самом деле это не дало бы мне ничего, кроме иллюстрации того факта, что ответ отдаляется от правильного ответа по мере того, как он становится более неправильно (пойди разберись).

            Итак, ограничения важны; то, что я только что описал, тривиально. Я их не понимаю. Я знаю, что исчисление часто используется для решения реальных задач, и что ограничения являются важным элементом исчисления, поэтому я предполагаю, что должно быть несколько простых реальных примеров того, что именно описывают ограничения.

            Какой простой пример ограничения в реальном мире?

            Спасибо

            -Хэл.

            • исчисление
            • алгебра-предварительное исчисление
            • анализ
            • пределы
            • приложения

            $\endgroup$

            1

            $\begingroup$

            Ваш пример предела – это предел, который легко оценить, но это вполне разумный пример!

            Вот еще один довольно простой для понимания пример предела, который позволяет избежать тривиальности.

            Если я буду подбрасывать монету столько, сколько потребуется, какова вероятность того, что я никогда не подброшу решку?

            Перефразируя как предельную задачу, можно сказать, что 9Н$. Тогда $$\lim_{N\to\infty}p(N) = 0$$ потому что $p=\frac{1}{2}, \frac{1}{4}, \frac{1}{8}, \frac{1}{16}, \ldots$ все ближе и ближе к нулю по мере $N$ становится “ближе к $\infty$”.

            $\endgroup$

            $\begingroup$

            Показание вашего спидометра (например, 85 км/ч) является пределом в реальном мире. Может быть, вы думаете, что скорость есть скорость, почему бы и не 85 км/ч. Но на самом деле ваша скорость постоянно меняется во времени, и единственные «твердые», т. е. «безграничные» данные, которые у вас есть, это то, что вы проехали 150 км от А до Б ровно за 2 часа. Цифра, которую дает вам ваш спидометр составляет в каждый момент $t_0$ вашего путешествия лимит $$v(t_0):=\lim_{\Delta t\to0}{s(t_0)-s(t_0-\Delta t)\over\Delta t}\ ,$$ где $s(t)$ обозначает расстояние, пройденное к моменту времени $t$.

            $\endgroup$

            $\begingroup$

            Мне трудно выйти за пределы математики в «реальный мир», поэтому позвольте мне привести вам этот «пример»:

            Пределы чрезвычайно важны, поскольку они служат основой для «производная» и «интеграл» — две фундаментальные структуры исчисления! В этом контексте пределы помогают нам понять, что значит «подобраться к точке сколь угодно близко» или «уйти в бесконечность». Эти идеи не тривиальны, и их трудно поместить в строгий контекст без понятия предела. Таким образом, в более общем смысле предел помогает нам перейти от изучения дискретной величины к непрерывной величине, и это имеет первостепенное значение в исчислении и приложениях исчисления.

            Чтобы применить это понятие к физике (да, я сейчас отхожу от математики), можно применить непрерывный анализ к движению. Мы хотели бы иметь возможность измерять мгновенных скоростей, что требует понятия мгновенных значений. Теперь это зависит от концепции предела. То есть мы хотим измерить величину в мгновении и определяем это «момент» пределом, т. е. как приближением к какому-то бесконечно малому времени. Вот как бы мы ответили, например, на банальный вопрос «с какой скоростью он двигался в момент времени $x$?». 9{рт} $$ хорошо известная формула непрерывного компаундирования.

            $\endgroup$

            2

            $\begingroup$

            Немного истории: Уже в 500 г. до н.э. были некоторые дискуссии о возможном существовании «бесконечно малого», т.е. существует ли мельчайшая частица времени, материи и т. д. Около -450 г. Зенон предложил некоторые парадоксы для обоих предположений. Чуть позже Евдокс предположил, что по крайней мере в математике следует разрешить рассматривать бесконечно малые числа.

            Википедия: Евдоксианское определение пропорциональности использует квантор «для каждого . 2$. И т. д. В конце Ахиллес так и не достигает Черепахи. Однако благодаря ограничениям мы можем показать, что Ахиллес должен пробежать только конечное расстояние, прежде чем он эффективно поймает черепаху. 9{n}}{1-\frac{v_b}{v_A}} \overset{n \to \infty}{\longrightarrow} d\frac{v_A}{v_A-v_b}< \infty $$ Таким образом, Ахиллес должен пробежать лишь конечное расстояние, прежде чем поймать Черепаху.

            $\endgroup$

            $\begingroup$

            Вот пример: Алекс участвует в спринтерском соревновании на 100$ миль, мы обозначаем время как $t$, расстояние как $F$, мы можем построить $F(t)=t\cdot V$ (предполагая, что скорость Алекса является константой вроде $10\ м/с$.) так каков предел $F$, поскольку $t$ приближается к $20$, мы легко можем увидеть $F(20)=200m$, это процесс ограничения. как это описать: когда t приближается к 20$, F приближается к 200$.

            $\endgroup$

            1

            $\begingroup$

            Чтобы двигаться по прямой из А в Б, вам придется достичь 1/2 точки C между A и B. Чтобы добраться из C в B, вам нужно будет добраться до середины линии CB.
            Продолжая движение на 1/2 оставшегося расстояния у вас всегда будет небольшая часть между вами и точка B. B называется пределом. Вы получите бесконечно близка к ней, но никогда не достигает точки B.

            $\endgroup$

            1

            $\begingroup$

            Когда дело доходит до реального мира, я обнаружил, что ограничения информируют нас о том, что нам нужно скорректировать наши « подъем » и « бег » (наклон), чтобы оставаться в пределах границы по мере приближения к ней. И для дальнейших практических целей пределы помогают нам положить конечное значение (асимптоту) в кажущееся бесконечном путешествии точности .

            Например, когда вы подъезжаете к знаку остановки. Вы начинаете нажимать на тормоз, и ваше ускорение со временем уменьшается, и вы замечаете, что это происходит, потому что вы видите, как ваш спидометр падает. По мере приближения к знаку «стоп» вы регулируете скорость, с которой падает ваша скорость, чтобы убедиться, что вы остановитесь в нужном месте. Вы начинаете замечать, что изменения в скорости становятся намного менее драматичными (но время, необходимое вам для продвижения вашей позиции, становится намного длиннее). Вы замедляетесь до такой степени, что становится очень трудно увидеть, снижается ли ваша скорость.

            Теоретически можно бесконечно приближаться к знаку “стоп”. За каждую единицу времени вы можете оказаться на половину расстояния ближе, чем раньше. Однако в какой-то момент вы говорите себе: «Этого достаточно, я считаю, что доехал до знака «стоп». Вы не хотите получать штраф за проезд через знак остановки, потому что обвинение в этом эксперимента в исчислении, вероятно, вам не поможет. Вы нажимаете тормоз до пола для полной остановки. Нулевой предел, с прагматической точки зрения, для вас и есть ноль. 92}$

            $\lim_{x \to 1} f(x) = \frac{x-1}{(x-1)\times(x+1)}$

            $\lim_{x \to 1} f(x) = \frac{1}{x+1} = \frac{1}{2}$

            , что является правильным способом. Это означает, что числитель и знаменатель будут приближаться к 0, когда x приближается к 1. Но на самом деле x никогда не достигает 1, и в действительности обе части рациона движутся с разной скоростью с течением времени. Таким образом, соотношение между ними будет $\frac{1}{2}$.

            $\endgroup$

            Пределы с пояснениями, практическими вопросами и ответами [Исчисление AP, Исчисление 101, Математика] – Moosmosis

            исчисление

            Ханна Гао на • ( 3 комментария )

            Расчет: пределы

            В этой главе мы познакомим вас с первой большой идеей AP-исчисления: пределы и непрерывность. Эта тема включена в курсы исчисления AB и BC.

            Даже если ваш курс еще не начался, хороший способ подготовиться к нему — изучить ограничения, поскольку их обычно легко понять. Это даст вам преимущество в изучении (скорее всего) первой темы, которую вы изучите в своем курсе AP, и может помешать вам отстать.

            Начнем!

            Лимит записывается в виде

            , где f(x) — функция. Это читается как «предел, когда x приближается к c от f от x».

            Здесь с — константа, но обратите внимание, что она может быть как бесконечной, так и отрицательной бесконечностью и будет записана как

            .

            Давайте посмотрим на наглядный пример. На приведенном ниже графике синий график представляет функцию. Посмотрите на точку x = 1, отмеченную зеленым цветом.

            Очевидно, что по мере приближения графика к x = 1 как с левой, так и с правой стороны точки функция все ближе и ближе приближается к значению f(x) = 0,

            Поскольку предел с обеих сторон точки приближается к одному и тому же значению f, мы можем написать

            Существует также особый способ написания границ справа и слева. Они называются односторонними пределами и записываются как:

            Обратите внимание, что предел, когда x приближается к f(x) справа, использует знак «+» в верхнем индексе рядом с c, а предел, когда x приближается к f(x) слева, использует знак «-» в верхнем индексе рядом с c .

            Зная, что односторонние пределы в точке определены и равны, мы можем заключить, что предел при приближении x к этому значению определен и равен тому же значению. Вернемся к приведенному выше примеру, мы можем написать:

            Поначалу это обозначение одностороннего предела может показаться бесполезным, но это потому, что мы рассмотрели только один пример, который был… ну, простым.

            Вот пример посложнее.

            Хм… этот график отличается от того, что мы видели. Первое, что мы замечаем, это наличие пробелов или разрывов в точках x = -2 и x = 2. Они называются разрывами скачков . Разрывы скачка возникают, когда оба односторонних предела в точке существуют, но различны.

            При x = 2 имеем

            А поскольку односторонние пределы не совпадают, нашего предела при приближении x к 2 не существует.

            Обратите внимание, что, хотя функция f(x) никогда не принимала значения 2,5 (на что указывает незакрашенный кружок в точке), мы все же могли выразить односторонние пределы в точке, поскольку пределы обозначают значение что функция приближается к , а не к фактическому значению, которое она принимает.

            Точно так же не существует предела при приближении x к x = 2 (еще один скачок).

            При x = 4 с нашей функцией происходит что-то странное. Значение f(x), кажется, становится все больше и больше без ограничений, когда x приближается к 4 слева, и кажется действительно безгранично отрицательным, когда x приближается к 4 справа. Мы называем это бесконечным разрывом , который возникает, когда оба односторонних предела приближаются к бесконечности или к отрицательной бесконечности.

            Мы обозначаем это как:

            Теперь давайте посмотрим, как определить пределы в точках без использования графика.

            Пусть g(x) = 2x + 1. Давайте посмотрим на значения g(x), когда x приближается к 3 слева. Другими словами, мы ищем предел, когда x приближается к 3 из g(x) = 2x + 1 слева.

            x g(x) = 2x + 1
            2 5
            2.5 6
            2. 8 6.6
            2.9 6.8
            2,97 6.94
            2.99 6.98
            2.999 6.998
            2.9999 6.9998

            I think you can figure out what would happen if we continue the table for values ​​of x которые очень близки к 3. Верно! Значения g(x) приближаются к 7, поэтому можно ожидать, что значения g(x) будут очень близки к 7.

            Точно так же, если вы посмотрите на таблицу со значениями x, приближающимися к 3 справа, вы увидите, что g(x) очень близко подходит к 7.

            Таким образом имеем:

            Вы можете проверить, что это так, построив график g(x) = 2x + 1 или просто подставив значение 3 вместо x. В любом случае мы получаем, что g(3) = 7, что подтверждает наш вывод о пределе.

            Однако не всегда верно, что предел, когда x приближается к некоторому значению c функции, равен оцененному значению c этой функции. В примере выше мы видели равенство, но это не всегда так. Мы можем только надеяться оценить значение функции в точке с, используя предел, и обычно мы получаем довольно хорошую оценку.

            ВАЖНАЯ ИДЕЯ:

            Если вам непонятно, почему это так, это будет иметь больше смысла, когда мы будем говорить о непрерывности .

            В нашем примере выше предел был равен оцениваемому значению при x = 3, потому что наша функция была непрерывной при x = 3.

            Пределы — очень полезные инструменты для оценки функций, и они широко используются в вычислениях. Хотя в этом уроке они кажутся довольно простыми, они могут стать более сложными и не всегда такими простыми. Иногда требуются манипуляции, чтобы найти предел в точке.

            Убедитесь, что вы полностью понимаете общую концепцию ограничений, прежде чем переходить к более сложным темам, поскольку они основаны на этой большой идее.


            Популярные статьи об исчислении

            https://www. mathsisfun.com/calculus/limits.html

            https://www.mathwarehouse.com/calculus/continuity/what-are-types-of-discontinuities.php

            http://tutorial.math.lamar.edu/Classes/CalcI/TheLimit.aspx

            https://www.analyzemath.com/calculus/limits/introduction.html

            https://www.khanacademy.org/math/ap-calculus-ab/ab-limits-new/ab-1-2/a/limits-intro

            Нажмите и ознакомьтесь с этими популярными статьями для получения дополнительной информации: 🙂

            Исчисление: две важные теоремы — теорема о сжатии и теорема о промежуточном значении

            Исчисление: пределы и непрерывность

            Популярные статьи об исчислении

            Эктодерма против Энтодермы против Мезодермы

            Психология 101 и мозг: стресс – определение, симптомы и последствия для здоровья реакции «бей или беги»

            Система кровообращения: путь кровотока через сердце

            Кровеносная система: структуры и функции сердца

            Артериальный проток против венозного протока против овального отверстия: кровообращение сердца плода

            Сердечные аритмии: определение, типы, симптомы и профилактика

            Верхние и нижние дыхательные пути: инфекции верхних и нижних дыхательных путей

            Семь основных функций дыхательной системы

            Анатомия пищеварительной системы: схема, органы, структуры и функции

            Эмбриология и развитие почек: простой урок

            Психология 101: Психология толпы и теория Гюстава Ле Бона

            Введение в эволюцию: Чарльз Дарвин и Альфред Рассел Уоллес

            Символика обуви в снах

            

            Поддержите нас на Moosmosis.

Оставить комментарий